ppr

Pataasin ang iyong marka sa homework at exams ngayon gamit ang Quizwiz!

After an informal observation, Mr. Miller receives written feedback from an assistant principal, Ms. Hill. Most of the feedback is positive, but Ms. Hill said that the end of the lesson needed improvement. What would be an appropriate response from Mr. Miller to this observation?

"Can we work together to develop a better method for ending a lesson?"

A fourth-grade parent walks with their child to the classroom at the beginning of the day. The parent is concerned about a grade on a project and asks the teacher to meet right then. Which of the following responses would be the most appropriate for the situation?

"I can see that you are concerned, but now is not the appropriate time to discuss this. I will contact you today to set a meeting time."

A math teacher is reviewing homework assignments and notices that one student is consistently making errors with the order of operations. Which of the following types of feedback would be most appropriate?

"It looks like you added before you divided. The order of operations is to solve for multiplication/division before solving for addition/subtraction. Let's try one together."

Mr. Bruin has noticed that a few of his ELL students have not been completing their classwork or homework assignments. He thinks they are unmotivated, but he's not sure how to increase their motivation. His colleague, Mr. Frank, suggests using a technique to increase their intrinsic motivation. Which of the following is NOT likely to be Mr. Frank's suggestion?

"Reward them with a prize, such as a class party, if all the students complete the homework assignments for a certain amount of time."

A group of high school students is taking a standardized reading achievement test. After the students receive their scores, the teacher meets with students to discuss how to interpret the results. One student in the class receives a score in the 80th percentile. How can the teacher explain this score to the student?

"Scoring in the 80th percentile means that you have performed as well as or better than 80 percent of high school students who also took this assessment."

Ms. Jolly projects a picture of the night sky from NASA for her students. One student, Pam, exclaims "The stars are colored!" Which of the following is the best response for Ms. Jolly to make if she is to use Pam's outburst as a teaching moment?

"Shall we find out why the stars have colors?"

Mr. Cates notices that his students are timid during group problem-solving activities. He wants to encourage his students to take more risks and demonstrate more creativity. Which of the following statements could he use that would best encourage this?

"There is no single correct answer to this problem, and every group could have a different, correct answer."

Which of the following teacher statements creates a positive atmosphere in which students feel supported and encouraged to meet the expectations set for them? Select all answers that apply.

"This is not the quality of work that you are capable of." "I am proud of you for trying your hardest."

The fifth-grade students at an elementary school host an annual cultural fair in the spring. Students present a project that displays various aspects of a chosen culture, including food, holidays, religion, and traditions. This project is more challenging than those students have completed in the past, and at the beginning of the year they have many questions and concerns. Which of the following comments from the teacher communicates high expectations for student performance?

"This seems overwhelming now, but we will work on the skills needed to help you create a successful project."

Which of the following teacher statements is an example of effective, positive feedback?

"This was a challenge, and you worked hard until you solved it!"

Which of the following statements describes an example of extrinsic motivation?

A student completes an extra credit assignment in order to earn a higher grade.

For which of the following problems would it be least appropriate to utilize a student-teacher contract as a solution?

A student punches another student out of frustration on the playground.

A test given at the end of a unit on weather and climate is an example of:

a summative assessment.

According to Jean Piaget's periods of cognitive development, students in upper elementary begin to apply which of the following?

logical interpretations

According to Piaget's theory on stages of cognitive development, which of the following thought processes best distinguishes a student at the formal operational stage?

A student creates a story about how winter would be better if snow were green and not white.

Which of the following best describes the benefits of ending each class period with a closure activity turned in as students leave the room?

Closure activities help the teacher to reinforce key concepts from the lesson and identify any gaps.

In which of the following situations would a Behavioral Intervention Plan (BIP) be appropriate for the student? Select all answers that apply.

A student is consistently late for school with different excuses. A student is always rushing through work and submitting incorrect answers with repeated careless mistakes.

Which of the following situations is an example of extrinsic motivation?

A student is motivated to complete her homework because her parents said that she can have ice cream once she is finished.

Mr. Hampton, a sixth-grade science teacher, is beginning an instructional unit and has planned the following instructional activities: two slideshow presentations followed by a group discussion and a worksheet, a classroom discussion, two experiments, and one field trip. The wide variety of instructional activities best demonstrates Mr. Hampton's understanding of which of the following principles?

A sixth-grade class represents a wide range of cognitive developmental levels.

Which of the following is not part of the fair use guideline concerning copyright laws?

A song may be used in its entirety, but cannot be edited or shortened.

Which of the following does not accurately reflect the 504 designation?

A 504 plan is the same as an IEP and they can be used interchangeably.

The human body has many parts that work together as a person develops. Which of these examples correctly expresses a delay in one area that impacts growth in another?

A child with developmental delays is socially delayed because he struggles to relate to peers.

A teacher knows that activating students' prior knowledge before learning about a new topic is a great way to engage students and improve their learning experience. Which of the following activities can she use to accomplish this? Select all answers that apply.

A concept map An anticipatory guide A brainstorming session

A teacher has assigned her 7th-grade class a research project focused on exploring different careers. Of the following options, which would be the most credible source for applicable information?

A forbes.com article comparing salaries and reported levels of happiness from different careers

Which activity would be most beneficial for students learning the structure and functions human body systems?

A project in which students run an election campaign for their assigned body system. Students write campaign speeches about why their body system is the most essential, make posters, and have a vote at the end of the project.

Which of the following would not be considered protected speech in an educational setting?

A senior history teacher discusses presidential candidates during class to attempt to persuade a newly 18-year-old student to vote for the candidate the teacher supports.

Conner is a sixth-grade student who consistently forgets to bring his homework and the necessary school supplies (paper, notebook, pen or pencil) to his classes. Repeated reminders and grade penalties have not changed his organizational skills, but he readily admits that he needs to get organized. His language arts teacher meets with him to develop a system and checklist for keeping track of his assignments. She has him report to her, at first daily and then weekly. What is the best reason that this plan would likely help Conner?

A system will help him learn how to monitor his own progress and performance more effectively.

The Code of Ethics of Texas Educators summarizes expectations for teachers personally and professionally. According to the code, can a teacher be held accountable for material posted on personal social media?

A teacher can be held accountable because the code refers to conduct both inside and outside of school.

Which of the following would show effective collaboration between a parent and teacher?

A teacher creates a behavior chart for a struggling student and the teacher and parent both sign it each day.

Franklin Middle School strongly advocates the use of positive reinforcement in its school. Which of the following is an example of positive reinforcement?

A teacher praises a student who correctly answers a difficult question.

A fourth-grade science teacher plans an experiment where students can freely manipulate the material, compare progress, and share their conclusions with their classmates. Which of the following is best demonstrated in the teacher's instructional strategy?

Active engagement in the instructional activities greatly increases students' learning ability.

Mr. Johns gave a test last week and Ginny missed one question. She answered that 14.5 people would ride on each bus rather than 15. Her parents would like a conference because she did the math problem correctly and should receive credit even though her answer was not reasonable. How should Mr. Johns handle this situation?

Agree to meet, listen to their concerns, and then explain that one component of math is understanding reasonable answers.

Which of the following is not a requirement when developing cooperative group work?

All students must have their own supplies.

Mrs. Dobbs is teaching students to skip-count by 2s, 5s, and 10s in her second-grade class. Earlier in the year, she evaluated her students learning style and assigns them one task based on this evaluation. Visual learners have been given a number line and they are to draw the hops across the top. Auditory learners have been given a list of the even number to 20, numbers divisible by 5 to 50 and numbers ending in 0 up to 100. They are told to say them over and over aloud to memorize the skips. Kinesthetic learners have been given a large number line on the floor. They are jumping to the next number as they skip-count. What can Mrs. Dobbs do to improve her teaching?

Allow all students to participate in all three activities by rotating through them.

A teacher is planning a long-term project for the next unit of study. She would like for students to take ownership of their learning. Which of the following will best help achieve this goal?

Allow students to choose their own topic or project.

Which of the following strategies best ensures a teacher will establish clear behavior guidelines?

Allow students to participate in the creation of classroom guidelines.

Ms. Nelson wants to ensure she is reaching all of her third-grade students and stimulating higher-order thinking skills. Which of the following is the least appropriate method to accomplish her goal?

Allow students to use any internet site for an independent research project.

Ms. Winski, a second-grade teacher, notices that her newest English language learner is struggling to communicate with his peers. Which option below would best encourage her new student to practice speaking to and interacting with his classmates?

Allow time before and after activities for students to chat with their deskmates.

Which of the following rationales would a supporter of learning management systems (LMS) provide to convince their school to adopt them?

An LMS can alleviate logistical burdens, like maintenance of student academic records.

A 10th-grade student is studying environmental science in school. For his research paper, he is writing about pesticide use in the modern-day food system. When writing his paper, which of the following sources should the student be wary of? Select all answers that apply.

An article published in a scientific journal in 1950 An internet blog post with an unknown author

A high school history teacher has just finished a unit of study and plans to assign an essay as students' final assessment grade for the unit. Which of the following best describes the reason an essay is the appropriate assessment instrument in this situation?

An essay allows students to organize their thoughts and demonstrate understanding of the topics addressed in the course.

A student is giving a presentation on various animal species found in the rainforest. She is planning to include images of each animal in her presentation to the class. In order to avoid any copyright issues, what type of images should the student use? Select all answers that apply.

An image from the public domain An image published under a creative commons license An image that she draws herself

Mrs. Jones wants to use a website in her presentation. Which of the following is the most important for her to do first?

Analyze the website and its content to ensure it is appropriate.

Ms. Wilson is having difficulty with new software introduced by the district. Which of the following is the best way to address this issue?

Any of the above.

When it comes to suspected abuse or neglect of a student the law in Texas dictates that teachers must:

make a report no later than the 48th hour after initially suspecting child abuse or neglect.

A new teacher is setting up her classroom and wants to ensure the space is ideal for learning. Which of the following is the least important question this teacher should ask when determining the layout of the room?

Are the colors matching in the space and does it look appealing?

Mrs. Clark finds that some of her students are not engaged in class discussion. Which of the following would be the most effective way to set up the space to encourage student engagement in class discussions?

Arrange desks in a circle so all students face each other.

Read the description of Lakesfield School and then answer the questions that follow. Mr. Logan is the principal at Lakesfield School. Lakesfield is the oldest school in Middlebrook County. The community is located about 25 miles north of a big city in a small semi-rural town, centered around a meat-packing plant. Almost a quarter of the students come from migrant families who work at the plant or in nearby farms. More than half of the students enrolled are eligible for free or reduced-price lunches. Approximately 40% of the students qualify as English-language learners. Many parents do not read or write well, but they have high hopes for their children and want them to succeed in school. Mr. Logan understands the importance of involving parents in their children's education, and plans to prioritize this in the upcoming school year. In past years, Mr. Logan has prepared an open house for parents at the beginning of the year, but attendance is always low. What three things should Mr. Logan consider doing this year to increase attendance? Select all answers that apply.

Arrange free or discounted transportation for parents who need it Hold two events, ensuring one is at a time when the meatpacking plant is closed Provide free childcare at the event for parents who need it

Leo, a sixth-grade student in Ms. Hane's class, has recently been acting out in class more than usual. He makes inappropriate comments and yells out incorrect answers to confuse students. Ms. Hane wants to initiate a Behavioral Intervention Plan (BIP). What should she do first?

Ask a parent or guardian for permission to screen Leo using a Functional Behavior Assessment (FBA).

Mr. Huff is a teacher in Florida. He would like to show his students how their daily activities have an impact on the ecosystem. Which of the following activities would be most appropriate to achieve this goal?

Ask students to choose a product they use every day and research the impact of this product on the ecosystem. Ask them to explain how production, use, and disposal of the product affects wildlife, habitats, and the wetlands around them. Have them create a brief presentation to show the class.

After a quiz over plant systems, Mr. Wyatt sees that most of his students believe that all plants cells have chloroplasts and chlorophyll. Which of the following activities would best correct this misconception?

Ask students to identify the function of chloroplasts and chlorophyll and whether all cells in the plant carry out this function. Have students view root cells under a microscope and tell whether or not these structures are present. Ask students to reflect on why.

Which of the following activities would be most suitable for an auditory learner during a unit on animal behavior?

Ask students to write a song or rap about different social systems and make a music video to go along with it.

A language arts teacher has asked students to evaluate an example of a persuasive text by critiquing or defending the piece in a small group discussion. Which activity should the teacher plan to include a task that increases the rigor of the assignment based on Bloom's Taxonomy?

Ask students to write their own persuasive text.

Mrs. Fischer would like to be sure her students know the implications of biotechnology and how it will affect society. Which of the following activities would be most appropriate for exposing students to a variety of perspectives?

Ask students who owns a cloned sheep: the owners of the sheep who provided the DNA, the scientists who created it, or the owner of the new incubating sheep? Lead students on a structured debate about this topic.

At the year's beginning, a new teacher becomes unsure about all of her responsibilities, including instruction and required documentation and record-keeping. Which of the following would be the most effective way for this teacher to secure help?

Ask the department head or team leader for advice and request a mentor teacher with whom to work.

An at-risk student has had multiple absences and tardies over the past month. What is the most appropriate way to help this student?

Ask the student if there is something preventing them from being at school and offer resources to overcome the obstacle.

Which of the following interventions would be most beneficial for a visual learner who is struggling to differentiate between DNA and RNA?

Ask the student to color a picture of the DNA molecule and RNA molecule. Ask the student to highlight the differences.

While giving a lecture, Mr. Wilhelm notices two students making a commotion and distracting the other students. Mr. Wilhelm is surprised because both students are typically attentive during his lectures. He verbally warns the two students to stop their behavior and pay attention. One of the students responds by saying, "This lecture is boring and I'm tired of paying attention." Mr. Wilhelm instructs this student to go outside the classroom and wait for Mr. Wilhelm to come out. Which of the following would be the best initial response when Mr. Wilhelm talks with the student outside the classroom?

Ask the student why he responded in the manner he did.

An elementary special education student complains to his P.E. coach that the work in his social studies class is too hard and that he just can't keep up. What is the best procedure for the coach to follow in order to advocate for this student?

Ask the student's special education teacher to intervene.

A science class is working on a laboratory activity. Their teacher is monitoring progress throughout the activity and notices that the two students' lab report shows that they have a misconception about the topic. Which would be the most effective action for the teacher to take?

Ask the students questions about their lab to encourage critical thinking to correct the misconception.

Mrs. Clark has tasked her students to write a poem about a memorable time in their life. Of the following, which is the most important assessment principle Mrs. Clark should implement?

Assessments should be objective and concrete.

A teacher notices that fewer students are completing their homework over the course of the semester and that average test scores have decreased. She has confirmed that the homework is congruent with instruction and would benefit the students. In an effort to help raise course grades, she would like to change the grading method of homework from completion grades to another method. Which of the following approaches will be most effective for encouraging students to complete their homework?

Assign 5-7 specific questions and grade while providing written feedback.

Which of the following should be the first step of a teacher who wants to establish a relationship with parents where both the teacher and parent can initiate a conversation to discuss the student's progress?

At the beginning of the year, set up a meeting with each parent to discuss mutual goals for their student.

When a new teacher begins at Franklin Park High school, they are always assigned a mentor teacher. The principal has been doing this for years because he knows that mentor teachers can do which of the following? Select all answers that apply.

Be a good resource for recommending articles or books to further the teacher's professional growth Provide ideas to new teachers about how to improve their instructional practice

The LEA has the responsibility to identify, locate, and evaluate all students who reside in the state of Texas who are in need of special education or related services, including students enrolling in private schools. At what age can children begin receiving services?

Birth

A teacher creates an assessment that includes questions at varying levels of understanding from basic memorization and recall to analysis and synthesis. Which of the following individuals researched these varying levels of understanding?

Bloom

When Miss Sanchez, a preschool teacher, resolves a dispute between two students, what is the best non-verbal tool to achieve an agreeable resolution between the two parties?

Bring herself down to the students' level to communicate.

Mrs. Greak just received her roster of second-grade students for the new school year. There is a "Back to School" event in two days, where students and families may come and meet their teacher. The principal has sent out an email to the school community informing them about the event and has asked her teachers to contact families as well. What is the best way for Mrs. Greak to reach out to her families?

Call each family to introduce herself and invite them to the event.

A teacher would like to increase student participation during a review activity. Which of the following strategies would best accomplish this?

Calling on students at random after providing ample wait time, and allowing peers to expand upon answers.

A teacher used the scope and sequence provided by the district to plan a unit prior to beginning instruction. After reflecting on the first week's progress, the teacher realizes that the students are not ready to move on to the skill she currently has scheduled for the following week. Which of the following best describes how the teacher should move forward in this unit?

Change the upcoming plans to meet the students' needs, but also revisit and revise the unit plan to ensure that all unit objectives will still be taught.

Joseph has ADHD. His teacher, Mr. Quinn, is aware of Joseph's diagnosis and knows that Joseph goes to the nurse to take his medication every day after lunch. One afternoon, Joseph is acting out more than usual and fidgeting in his seat. Mr. Quinn suspects that Joseph has forgotten to take his medication. What should Mr. Quinn do?

Check with the school nurse to see if Joseph has taken his medication.

A typical 5th-grade student should be capable of: Select all answers that apply.

Classifying and grouping objects. Manipulating objects to test out ideas. Understanding varied perspectives.

Mrs. Miller, a sixth-grade science teacher, is striving to create a classroom environment that encourages learning and academic effort. She does not want to have a classroom culture that causes stress and anxiety in the students. Which of the following would best accomplish Mrs. Miller's goal?

Clearly outline expectations for student behavior and performance.

A middle school English language arts teacher regularly uses the instructional reading strategy K-W-L when introducing a new unit of study. Students have a section of their notebooks dedicated to K-W-L responses, and the teacher collects the notebooks once a month to review their entries. Below are K-W-L entries for two students following an introductory lesson on poetry. Student 1: I know that poems usually rhyme. I want to know how to understand a poem, especially if it has new words. I learned that haikus and limericks are two types of poems. Student 2: I know that poems use imagery and vocabulary words to help the reader understand the poem. I want to know how to write poetry that others want to read. I learned what stanzas are. The teacher would like to expand upon the K-W-L strategy in a way that would be most beneficial to the group of students. Which of the following would be the best way for the teacher to achieve this goal?

Collecting notebooks more often and adding comments in response to the students, then providing students with small group learning activities that focus on areas in which they want to learn more.

A teacher notices that many parents of English language learners struggle to comprehend the purpose, content, and rating systems of state tests. Which of the following actions should the teacher take to most effectively address the needs of the parents?

Conduct a meeting for parents about state testing, and ensure that bilingual staff is available to translate the information if needed.

Mr. Williams is planning a research unit. He remembers feeling disappointed in the revision step of the writing portion the previous year. The first draft was due just a few days before the final draft, and very little revision was actually completed by the students. This year, he wants to prioritize completing a rough draft of the essay early enough to provide plenty of opportunity for revision. Which of the following would be the most logical way to achieve this goal?

Consider which topics and skills could be taught after the rough draft is completed, to then be used during the revision process to improve the essay.

A student finds a chart on a website while doing research and wants to include the chart in her research paper. What should she do?

Copy and paste the chart in the paper, citing the source.

Which statement below falls within the highest category of Bloom's taxonomy?

Create a commercial advertising your favorite figure of speech.

Wilmer Middle School has invited more parent volunteers to help the classroom teachers. By increasing the number of parent volunteers present in the school's classrooms, the school is most likely to achieve which of the following?

Create a greater awareness in the school community to help achieve important student goals.

A first-grade student frequently disrupts class discussion, has an unorganized desk, misplaces his classroom materials, and has a hard time maintaining eye contact with whom he is speaking. His teacher becomes concerned, as these behaviors are causing him to struggle in his academic studies. Before she contacts the student's parents to schedule a meeting, which of the following is the most appropriate first step?

Create and gather documentation about specific instances of the student's behavior.

Mr. Oden would like to develop a set of classroom behavior standards to create a positive environment. Which of the following strategies would be the best way for Mr. Oden to accomplish this goal?

Create classroom rules during a group discussion with students, where they can provide their input and opinions for behavioral consequences.

Students in Mr. Clark's classroom have a wide range of academic ability levels. Mr. Clark wants to regularly use group activities to support the classroom learning objectives. Which of the following strategies is most likely to help promote the success of all students during these group activities?

Create heterogeneous groups with a defined goal, where all students have a specific, unique role in achieving the goal.

An elementary classroom has a new student. The teacher realizes it can be a slow process for the new student to be accepted and feel welcomed in a new classroom. Which of the following strategies would best promote the class's acceptance of the new student?

Create teams of students and assign each team to help the new student learn a classroom activity and daily routine.

Mr. Williams is using backward design to plan the research unit. Which of the following would be the most logical first step in this planning process?

Creating a rubric for the final project that corresponds to the state standards.

Mr. James plans to assess his students' knowledge during a unit on linear functions and would like feedback from the students on how well they feel they are learning the concepts. Which of the following assessment would be the most appropriate for Mr. James to use during the unit?

Daily open-ended formative assessment in which the students complete a problem, with justification and any questions they still have about the material from that day.

Of the following, which would be most beneficial to students using the Internet for research purposes?

Demonstrate the most efficient ways to search for topics, identifying the best words to use and how punctuation can influence search results.

A middle school has recently decided to implement a change in the bell schedule which will allow students a 30 minute "free period" two days per week. During this period, students must be in a classroom but may work on enrichment activities, participate in remediation or study skills, make up tests and assignments, or participate in other activities at teachers' recommendations. Department leads are working with school administrators to create guidelines for the new program. Which of the following is the first step that they should take to ensure the success of this program?

Develop a set of goals for the program and determine how they will track progress toward meeting these goals.

A high school teacher has a class that includes students with disabilities as well as English Language Learners. The teacher is preparing to begin a unit of study on a novel and has outlined the following activities: Activity 1: Review common fiction genres and take the students through a review of the novel's title, cover page, and summary to identify the genre. Activity 2: Review elements of plot and characterization. Provide the students with graphic organizers to use while reading the novel. Activity 3: Have students begin reading the first three chapters independently while using a graphic organizer to identify main characters and elements of characterization. Activity 4: Have students work in pairs to discuss what they have learned about the characters and plot during the first three chapters of the novel. The teacher wants to ensure that all students are grasping key concepts while reading the novel. Which of the following describes the best way to accomplish this?

Developing regular check points in which the teacher assesses students' understanding and adjusts instruction accordingly.

Mrs. Pontian is concerned about Julio because his fine motor skills seem to be lacking compared to the other students in her class. He struggles with using scissors and holds a pencil with an odd grip. What should be her first step in addressing this issue?

Discuss the concerns with Julio's parent.

During a lesson about cloning, a student interrupts the teacher and mentions a news report about scientist in China working on cloning technology. Which of the following is the most appropriate and immediate response to the interruption?

Discuss the connection between the news report and the lesson.

If Mr. Melton desires to evaluate the learning environment in his classroom, which of the following would be the most important question to ask himself?

Do I consistently address all my students' intellectual involvement and active learning preferences when I design my lessons?

Mrs. Hill notices her students have a hard time staying focused as she walks around the room answering the students' questions. She knows the students need to stay focused to maximize their learning. Which of the following would be the best strategy for Mrs. Hill to implement?

Establish and maintain classroom procedures for student behavior.

Which of the following would be the most effective way to organize a classroom to help a first-grade student with a hearing impairment participate in daily classroom activities?

Seat the student close to instruction so that he can look at the teacher when she is speaking.

An 8th-grade history teacher is teaching his students about past presidents. As part of the unit, he plans to have a mock election, in which each student acts as a former president and presents a persuasive campaign speech. The following steps will be performed over a week. Step 1: Students will be shown a video clip of two presidential nominees giving a campaign speech. Students will be asked to share their opinions on the candidates. The teacher will reveal who the candidates are and who won the election. Step 2: The teacher will present the elements of a persuasive speech. The students will watch another video and identify the elements within the campaign speeches. Step 3: Students will be asked to name some former presidents and their accomplishments. Students will complete a graphic organizer matching each president to his major accomplishment. Step 4: Students will be assigned a president to research and act as when giving a campaign speech. The teacher will randomly select sets of two presidents to compete against one another in a modern-day mock election. Step 5: Students will research their president and prepare a 2-3 minute campaign speech, addressing the issues of today, for a mock election. Additionally, students will prepare a campaign slogan and poster. Step 6: Students will give their speeches and classmates will vote for a winner. Which of the following questions could the teacher ask to introduce the topic and elicit student interest before describing the task to students?

Do you think that past presidents could handle the current issues in America today?

A teacher is setting up the classroom at the beginning of the school. The teacher will have two students with physical impairments. Which of the following questions is the most important point for the teacher to consider?

Does the classroom layout allow students to easily navigate the room?

Mr. Herman is a new principal at Hardy Elementary school. During his training period, he learns that Hardy Elementary school has a 25% new teacher attrition rate. Mr. Herman, shocked that this number is so high, vows to provide more support and encouragement to new staff. To do this, he plans to: Create a detailed teacher orientation program to assimilate new teachers to the school and community culture Develop a high-quality teacher mentoring program to support, guide and encourage new teachers Dedicate more resources to collecting teacher feedback at consistent points throughout the year Develop school committees in which teachers can collaborate with one another Seek ways to be more progressive and provide ongoing support As part of the mentoring program, Mr. Herman makes an info sheet that provides examples of questions that the new teachers can ask their mentor teachers. Which of the following questions should not be included on this list?

Does the student have a difficult home life?

Paul, a student in Ms. Vail's 8th-grade class, hasn't been completing his recent homework assignments. When asked for a reason, he informs Ms. Vail that he doesn't have Internet access at home, and his parents have been working late. Paul doesn't have Internet access at home, and the assignments have required some extra Internet research. What can Ms. Vail do to best support Paul?

Don't penalize Paul for the missed homework. Instead, work with Paul's parents and school administrators to ensure that Paul has the materials he needs to complete his homework.

Which of the following are examples of a point at which an informal assessment is appropriate? Select all answers that apply.

During a lesson; after introducing a new topic At the beginning of a unit

ELLs must simultaneously learn a language and gain knowledge in core content areas. Recent studies indicate that because of the additional challenges they face:

ELLs may do twice as much work as the native English speakers in the same school.

Which of the following appropriately identifies the teacher's role in understanding and using technology?

Effective teachers should understand basic terms and concepts related to current technology. Most classrooms have technology for teacher and student use and should be utilized appropriately.

Which example below reflects a student who is intrinsically motivated?

Emily practices the french horn for an hour each night because she enjoys improving her skills and learning new songs.

Megan, a seventh-grade student at Wilmore Middle School, consistently has trouble remembering and completing assignments. Her teacher, Mr. Johansson, schedules a parent-teacher conference to discuss Megan's situation. Mr. Johansson outlines the class routines, discusses how they help students remember classroom assignments, and explains how he can help support Megan by helping her remember assignments. Mr. Johansson then asks Megan's parents for support at home. Which of the following would be the best action for Megan's parents to help Megan at home?

Encourage Megan to find a consistent, quiet place and time to study. They can check with her daily to monitor how well she is writing down and remembering assignments.

A high school English teacher gives students 15 minutes to read at the beginning of each class period. After reading, students respond to short prompts in a writing journal. Many of the students are reluctant readers and often struggle to start the activity. Which of the following describes the best way a teacher can encourage participation?

Encourage students to choose texts that interest them including short stories, magazines, or graphic novels.

At Monroe High School, the teachers are taught to act as facilitators in the classroom. In order to fulfill this role, teachers are encouraged to do which of the following? Select all answers that apply.

Encourage students to problem solve and find answers for themselves Provide students with resources to guide their learning

Which of the following advocacy strategies is recommended for increasing family and community involvement with ELLs?

Encouraging parents to read and tell stories to their children in their native language.

Carmen is a third-grade student who routinely answers the teacher's questions without being called upon by the teacher. This is detrimental, as other students are not given the ability to participate because of Carmen speaking out of turn. Of the following, which is the best option to ensure Carmen develops the patience to wait to be called upon by the teacher?

Establish a consequence for each time Carmen speaks without being called upon and consistently enforce the consequence.

Ms. Lavine plans a lesson with the following learning outcome: "Students will write an introductory paragraph with an engaging opening statement, two supporting sentences, and a topic sentence." After teaching and modeling the creation of an introductory paragraph, Ms. Lavine gives the students some time to create their own introductory paragraphs. Once completed, she pairs the students for a peer conference on one another's writing. She explains that it's important for students to focus the feedback on the lesson objective and instructs them to clarify what parts of the writing are strong and explain why other parts might need to be strengthened. The students then form pairs and proceed to critique each other's work. In order for the students to provide feedback in a productive manner, which two of the following things should Ms. Lavine do? Select all answers that apply.

Establish clear criteria which students can use to evaluate each other's work Pair herself with a student to model the type of feedback she would provide

A third-grade teacher is planning a small project during social studies. There are multiple steps that must be completed in order. How can the teacher best communicate the steps to the students?

Explain one or two steps at a time and help students complete those steps before going on.

Mr. Ortiz is a paraprofessional that provides in-class support for sixth-grade students with emotional disabilities. The students will receive five minutes of free choice time if they complete work independently. Mr. Ortiz often prompts the students to begin working and reminds them of their free choice activities, but he doesn't consistently follow through with providing free choice options before moving onto the next activity. How can his supervisor help with the implementation?

Explain the effects of assigning consequences and not following through on them, especially with students who have behavior concerns.

Mr. Muldoon is teaching about converting between metric and English units. One of his students refuses to engage in the activity and states "I am never leaving America so I don't need to learn this." How should he respond?

Explain the many ways in which people in America use the metric system.

Which of the following is the most effective way to increase participation from students who are reluctant to volunteer to answer questions?

Inform students that everyone will be asked to answer a question, and then randomly call on students after sufficient wait time has elapsed.

Why is it important for schools to familiarize parents of English language learners with aspects of the US school system such as how to interpret report cards and assist students in creating their schedules and selecting elective classes?

Familiarity with the school system enables parents to collaborate more effectively with the school staff.

Which of the following requires that schools keep student information and records confidential?

Family Educational Rights and Privacy Act (FERPA)

Which of the following statements best describes a formative assessment?

Formative assessments measure what students know along the way.

The following characteristics are elements of which component of IDEA? Instructional services designed to address the educational needs of students with disabilities as comparable to nondisabled students Instructional services to the maximum extent possible as to meet the educational needs with nondisabled students Standardized and established due process procedures Evaluation, placement, and reevaluation established procedures to ensure appropriate placement and classification of student with disabilities

Free Appropriate Public Education (FAPE)

A few times a year Ms. Preston separates her second-grade students into heterogeneous groups for project-based work. She notices that whenever she does this, the same students always take control and do the majority of the work. What can Ms. Preston do to stop this from happening?

Give clear roles and responsibilities to each student in a group.

Which of the following is the best way to help students understand the scientific process when they are learning about potentially controversial topics such as the origin of the universe?

Give students time to discuss and form opinions from evidence.

Mr. Washington is teaching a new mathematics concept to his ninth grade class. He observes that some students have already mastered the concept while other students are having difficulty understanding the new concept. Which of the following would be the best option to help students struggling with the new concept while keeping the entire class engaged?

Group students into cooperative learning groups, and then those students who have mastered the concept will be able to help those students who do not understand the respective concept.

Which of the following does not accurately describe an aspect of physical development during adolescence?

Growth slows, but proportions become less childlike and more like an adult.

Mr. Popov is teaching his second-grade class about the benefits of the metric system. Which of the following activities is the most effective way to convey its ease of use to second graders, as compared with the U.S. customary system of measurement?

Hand out objects of varying length and ask students to measure the lengths in centimeters and in inches. Discuss the results.

Which of the following is not a specific and measurable question a teacher should ask when creating goals for improvement?

Have I allowed students to be engaged and involved in the learning process?

A teacher is reflecting on their practices and wants to evaluate how well they have ensured educational equity for all students. Which of the following questions should they ask themselves?

Have all of my students been able to access all parts of the curriculum?

A third grade teacher is planning a new geometry unit based on the state curriculum. Which of the following questions should the teacher ask to ensure an effective first lesson?

What is the extent of the students' prior knowledge of geometry?

Following an exploratory science activity outside, a prekindergarten teacher transitions her class into center time. The students are having issues with sharing, begin arguing, and are moving constantly from center to center. What strategy can the teacher use to redirect her classroom?

Have all of the students pause what they are doing and quietly review the expectations for center time.

Before the start of her first year of teaching, a sixth-grade teacher with multiple class periods decides to survey her peer teachers for ways to organize her classes so that appropriate routines and procedures are effective during group work. The experienced teachers offered several suggestions. Which of the following would be the best way to address this important need?

Have desks put together in groups of six, and provide labeled color-coded baskets in which each class period is to submit their assignments.

A teacher wants her fifth-grade students to learn that some tasks require persistence to complete, and to experience the joy from working hard and accomplishing a goal. Which of the following would be the best strategy for accomplishing this?

Have each student work on a research project with various checkpoints throughout the assignment to monitor their progress.

Weather maps can be quite complex, often showing wind speed, isobars, fronts, and precipitation. Which technique is most likely to enable upper elementary students to begin to learn to read weather maps?

Have students interpret several maps, one at a time, which each show only one type of weather symbol.

Ms. Babu feels her class has a good understanding of the definitions and theories associated with physical and chemical changes, but she wants them to apply the concepts. Which homework assignment would best help students connect the concepts to their everyday lives?

Have students look for a chemical or physical change they see after they leave school, and then tell the class about it the next day.

Some of Mr. Lynn's students are struggling to differentiate between the embryo and the fetus. Which of the following activities will best help the students clarify this information?

Have students pair up, research the differences, and make a Venn diagram or other graphic organizer of the information. Have them share their findings with the class.

A teacher would like to assess students' understanding of the similarities and differences between mitosis and meiosis. Which of the following would be the most effective way to do this?

Have students partner up and create a Venn diagram for mitosis and meiosis.

Mrs. Hurd is teaching a unit on the skeletomuscular system to a class that consists mostly of athletes and students who enjoy being active. Which of the following activities would most help them draw connections between the course material and their daily lives?

Have students perform a fitness test and review how the skeletomuscular system helps them perform each task in the test.

Which of the following is a learner-centered method of explaining a game to a student?

Have students try each step of the game as the teacher explains.

Mr. Jeffords would like to show his students that all living things grow and develop and that metamorphosis is a process some animals go through as they grow and develop. He knows that he has several kinesthetic learners in his class. Which of the following activities would best address the needs of these students?

Have students write and perform a play to act out the process of a caterpillar changing to a butterfly.

Mr. Gwynn finds that only one or two students answer when he asks the class questions about the effects of land on climate. Which of the following is the best technique for getting more students to respond?

Have the students pair up, ask the question, let students discuss the answer with their partner, then ask the question again of the entire class.

Which of the following is the most appropriate assignment for helping early elementary students begin to consider their own effect on the environment?

Have them look for and talk about ways they and their family pollute the land, water, or air in everyday life.

Students in second grade are learning about metamorphosis. What is the best way for them to learn this concept?

Have them raise a caterpillar and document each day what they observe as it forms a chrysalis and becomes a butterfly.

Which of the following strategies helps to build positive, trusting relationships with students and, consequently, leads to the development of a positive learning environment? Select all answers that apply.

Having conversations with students about their personal interests Being a good role model for students Implementing fair and consistent discipline policies

During a heated debate in Mr. Martinez's twelfth-grade government class, a student declares that the problem with government spending is that the Congress spends so much money on themselves that they became unaware of the value of a dollar. Mr. Martinez then addresses the class in a calm voice saying: "It has been suggested that the incentives of Congress voting themselves pay raises and benefits results in a problem of fiscal irresponsibility of the government. Who agrees or disagrees with this statement?" Mr. Martinez's rephrasing of the student's declaration is most helpful because:

He is encouraging students to apply higher-order thinking to real-world problems.

Mr. Watson's seventh-grade Language Arts class is completing the unit on grammar, and he has noticed that several students in the class who normally excel are not completing homework. In order to encourage them to complete the unit with the same engagement as the beginning, he offers classes with 100% homework completion a reward. Why would Mr. Watt make this decision?

He knows that incentives can help students who have lost motivation to complete tasks they find less interesting, and he strives to create a positive classroom assignment.

Mr. Geller's students are studying earthworm responses to soil moisture content. They set up an experiment where they place earthworms in the middle of a pan of soil, with half the soil moistened and half dry. Mr. Geller hears some of his students discussing how the earthworms must be heading toward the wetter soil because they are thirsty. What should Mr. Geller's response be in order to support critical thinking?

He should ask the students leading questions to get them thinking about how earthworms breathe, which stimulus they are responding to, and why they need an environment with more moisture

Rachel, one of the students in Mr. Oliver's 9th-grade class has been asking to use the bathroom every day during class. Today when she asks, Mr. Oliver watches her walk down the hallway; he sees her take out her cell phone and begin texting. Lynnbrook Middle School has a strict no cell phone policy; any cell phone that is found on school grounds is confiscated and reported to the school administration. When Rachel returns to class, Mr. Oliver asks her to step outside for a minute and tells her what he witnessed. Rachel claims that there was a family emergency. Mr. Oliver asks for the cell phone and checks her messaging history to find that her most recent texts were to another student at school. What should Mr. Oliver have done differently?

He should not have searched in Rachel's messaging history.

During an instructional unit, Mr. Ramirez gives a pop quiz to his students. The results of the quiz are upsetting to Mr. Ramirez, as over half the students do not pass. Mr. Ramirez consults with his mentor teacher and they decide the questions are clear and that they properly address the material that has been taught. Which of the following is the best question for Mr. Ramirez to ask himself?

How did the previous instructional technique not adequately ensure material retention, and what changes can I make in the future to increase the students' understanding?

During the summer, a teacher obtains the standardized test results of the previous year for her upcoming students. This information can be best used to:

Identify the instructional needs of each student and plan instruction to address gaps.

Child Find is a section of the Individuals with Disabilities Education Act (IDEA) that requires local education agencies to:

use appropriate channels to locate, assess, and provide services to children with disabilities.

The parents of a student with disabilities have complained that their student is not getting the services they need. Which of the following statements accurately describes the rights of students and parents in a dispute with the school?

If they cannot reach an agreement with the school or school district, they are guaranteed due process to resolve the dispute.

Which activity would best help a kinesthetic learner understand the model of an atom?

In a large open space, have students hold signs with "+", "-", or "0" on them. Group students so all "+" and "0" signs are clustered together in one place. Have all students with "-" signs run randomly around the central group.

While an eighth-grade team researches the campus recycling practices for an interdisciplinary project, the students notice that a great deal of trash can be seen in various spots around campus. Which of the following is the best example of the teacher using this teachable moment to support eighth-grade curriculum?

In addition to developing their recycling plan recommendations, the teacher asks the students to analyze the trash problem and present possible student-involvement solutions. The group then must write a letter to the school's administrators, outlining how this solution could be implemented.

Ms. Pepper is using discussion to stimulate higher-order thinking skills after the class has read a story. Which of the following questions would be the best choice to accomplish this goal?

In the book Hatchet, Brian has one tool that helps him survive the wilderness. What is one item you would bring in a similar survival situation and why?

A high school history teacher has noticed that her students are showing a lack of interest in the subject. They enjoy debating why historical things take place, but the students seem to believe that the past has little bearing on their current lives. What could the teacher incorporate to increase student interest?

Include a current events question at the beginning of class that can be tied into the material.

During a math lesson, a teacher notices that many students are doodling on their notes page or fidgeting in their seats. Which of the following is the best instructional approach to the situation?

Incorporate a small group activity that requires students to move

Mrs. Worthington, the parent of a first-grade student, approaches her child's teacher because she is unhappy with the instructional methods of the teacher. Mrs. Worthington believes the class needs more rigorous standards in assessment and that there is a lack of structured learning in the classroom. Mrs. Worthington is concerned the students are not being adequately prepared for the higher grades. How can the teacher best respond to Mrs. Worthington's concerns?

Inform Mrs. Worthington of how the class activities promote the learning objectives while preparing the children for the more structured environment that they will experience in the higher grades. Allow her to observe the class and discuss with you any concerns she has thereafter.

As a warm up, a teacher has asked students to work in pairs to fill-in the states onto a blank map of the U.S. While they work, the teacher walks around the room, observing student progress. Which of the following describes what the teacher most likely trying to do?

Informally assess student mastery of the state names, and use the data as a formative assessment to guide instruction.

In planning a morning schedule for kindergarten students, which of the following guidelines should the teacher take into account?

Instructional activities should be balanced between allowing students to be active during instruction and requiring students to be still and listen.

A six-year-old kindergartener has shown difficulty raising her hand to share responses during direct instruction. The student's teacher has chosen to reward the student for raising her hand but does not provide a reward on each occasion when the student raises her hand. This is an example of

Intermittent reinforcement

A seventh-grade teaching team plans instruction that could involve language arts, math, science, and social studies objectives. They decide to have students collect data related to school and community recycling and design and implement a plan to address identified needs. What is the best developmental reason for selecting this approach?

It recognizes students' increasing ability to develop abstract thinking and focus on the world beyond the school setting.

Mr. Marks wants to create a class-wide research project for his third-grade class about the school's water usage and how the school can conserve their resources. As he discusses the project, the students become excited about being able to research various aspects of the school. Mr. Marks decides to split the class into small groups with each group coming up with ideas to implement the research. Which of the following is the most important advantage of incorporating student input into the planning of the project?

It allows students to help structure and determine their own learning.

A fourth-grade teacher plans a science unit that includes two experiments, one class project, a teacher presentation, and a field trip. Which of the following is the most important advantage of the teacher's instructional unit?

It allows students with varied learning styles to process the information using multiple sources.

Every Friday, a teacher leaves a note of encouragement on each of his students' desks, and he encourages the students to do the same for their classmates. What is the primary purpose of this?

It cultivates a positive classroom environment.

Mrs. Adams is preparing a lesson in which she will ask students to compare a portion of a book they are reading with a clip from the film version of the book. The clip has been chosen because of its unexpected interpretation of the text. On the board, in the section dedicated to the day's agenda, Mrs. Adams writes, "Determine the success of a film clip in interpreting the author's depiction of setting and characterization." Which of the following best describes the purpose of this sentence in the lesson?

It describes the learning objective for the lesson.

Every year on the first day of school, Mrs. Lavigne works with her 4th-grade students to create classroom rules, outlining their expected behavior for the year. What is the purpose of involving students in this process?

It encourages self-monitoring of the rules.

A new teacher and a mentor teacher agree to meet every other week. The meetings begin with the new teacher sharing one thing that is going well in the classroom and one thing that they would like to improve upon. Which of the following is the greatest benefit of beginning the meeting this way?

It ensures the meetings are as productive as possible.

Mr. O'Leary is establishing the classroom rules and procedures for his eighth-grade class. The first day of school, he posts the rules and procedures and invites students to comment, question, or propose new rules and procedures for the class. Which of the following describes the primary benefit of this activity?

It helps students feel a sense of responsibility and ownership in the rules.

Mrs. Kirsten is an eighth-grade language arts teacher at McNabb Middle School. Each day at the end of class, she has students spend ten minutes journaling about their views of the characters and situations of the current book they are reading. Which of the following describes the greatest benefit of journal writing for students?

It helps students reflect on new concepts and evaluate their ideas on what they are reading.

A high school ESL teacher invites a multilingual speaker to the classroom once a month to speak about their career and answer student questions. This month the speaker is a psychologist. In her presentation she mentions how being multilingual has helped her to be a better communicator because of her ability to see other people's perspectives. What is a primary benefit of her mentioning this?

It helps students see an advantage of being bilingual.

Every day during recess, Mr. Hernandez observes his third-grade students. He notices the games they are playing, the other students they are interacting with, and what activities interest them. Afterward, he always pulls a student aside to compliment them on a skill they showcased or speak about something he noticed them doing. What is the main purpose of this?

It helps to improve his relationship with the students.

According to research, why is increased wait time after a teacher asks a question an effective instructional tool?

It improves student responses and questions.

Every day when students arrive at Mr. Johnson's science class, he has a question or two written on the board. The question is based on something that they have learned the previous day and leads into what they will focus on that day. What is the primary purpose of giving students this task to complete at the start of class?

It maximizes classroom instruction time.

An 8th-grade history teacher is teaching his students about past presidents. As part of the unit, he plans to have a mock election, in which each student acts as a former president and presents a persuasive campaign speech. The following steps will be performed over a week. Step 1: Students will be shown a video clip of two presidential nominees giving a campaign speech. Students will be asked to share their opinions on the candidates. The teacher will reveal who the candidates are and who won the election. Step 2: The teacher will present the elements of a persuasive speech. The students will watch another video and identify the elements within the campaign speeches. Step 3: Students will be asked to name some former presidents and their accomplishments. Students will complete a graphic organizer matching each president to his major accomplishment. Step 4: Students will be assigned a president to research and act as when giving a campaign speech. The teacher will randomly select sets of two presidents to compete against one another in a modern-day mock election. Step 5: Students will research their president and prepare a 2-3 minute campaign speech, addressing the issues of today, for a mock election. Additionally, students will prepare a campaign slogan and poster. Step 6: Students will give their speeches and classmates will vote for a winner. In Step 6, the teacher has the students agree upon some rules for the mock elections. The class agrees to cast their votes in private, by writing their choice, along with a reason, on a sheet of paper. Why does the teacher have the students do this?

It promotes a positive classroom climate.

A third-grade teacher plans to integrate writing, speaking, and the arts with reading instruction. Students select a favorite character from a book or story, write a few lines of dialogue that demonstrate what the character is like, design or bring a simple prop, and present their character to their classmates. What is the best reason for this instructional design?

It provides integrated, active learning and play that support development.

What is the primary benefit of a teacher working with a student to think about the step-by-step causes of a problem and the possible solutions?

It sets an example for problem-solving strategies that the student can use in multiple situations.

Every other month an elementary school holds Family Literacy Nights, during which working parents are invited to the school to participate in a night of reading with their children. Parents and children can join a read-aloud circle, watch as teachers demonstrate echo reading, or read with their child separately in another part of the room. There is an activity corner where parents and children can create bookmarks while enjoying snacks and refreshments. What is the main benefit of an event like this?

It teaches parents ways to support literacy development at home.

A new student, Julia, has started at Monett Elementary and the teacher is concerned about her social development. Which of the behaviors below are causing concern?

Julia sits alone at lunch and plays by herself at recess even when invited to join a game with others.

A middle school teacher has noticed many students in her classroom have formed cliques and several students are being excluded from these social groups. The excluded students have demonstrated a decrease in their willingness to participate in classroom activities and group discussions. Which of the following strategies would be the best first step in rebuilding a positive learning environment for all of the students?

Lead a group discussion about cliques and how it feels to be included and excluded from activities.

Mr. Klement's seventh-grade class is studying renewable and nonrenewable resources. He would like to encourage the students to apply their knowledge of resources to a real-world scenario and make an ethical judgment. Which of the following activities would best allow him to do this?

Lead students in a debate about whether individuals should be required by law to use renewable energy sources.

A high school world history teacher has students use journals several times a week to answer reflection questions, either at the beginning or end of class. The following are sample questions students are asked to respond to in their journal. What was the main concept in the news clip we watched at the beginning of class? How is that news story related to what we studied today? What were the two best ideas that came from your discussion group? Which of the following learning principles best reflect the teacher's use of these questions?

Learning is reinforced when students have opportunities to actively process and reflect on new information.

Which of the following expresses how a behavioral theorist views learning?

Learning occurs as a result of processing information and making associations between stimuli and responses.

Mrs. Buff receives a letter from the mother of one of her fourth-grade science students about the amount of time it is taking to do the homework for her class. Mrs. Buff has been working with all her students on developing appropriate study skills, including setting timelines for homework. Mrs. Buff responds to the letter from the parent with an invitation to the mother and her husband to meet with her concerning this issue. Which of the following would be the best strategy for Mrs. Buff to begin the conference?

Listen to the parents' concerns and then explain the study skills that have been taught and discussed in her classroom.

A school counselor has arranged a parent-teacher conference for a student who is experiencing an increase in behavioral difficulties. After introductions from the teachers, the parents are given an opportunity to express their concerns. The parents list a series of events in which they feel that their child has been unfairly targeted by school staff. Which of the following describes the most appropriate way for the staff to address the concerns of the parents?

Listening patiently as parents share their concerns and have a member of the team summarize the concerns of the parents.

The basic needs of a human, such as hunger and thirst, must be met before a person can move onto more complex needs such as safety, belonging, and self-confidence. These ideas are the work of which theorist?

Maslow

An elementary school noticed students were coming to school hungry and having difficulty focusing on classroom activities before lunch. The school put in place free breakfast for all students at the beginning of each day and saw a great improvement in student ability as a result. Which of the following learning theorist's ideas does this example best reflect?

Maslow (hierarchy of needs)

When Mr. Troutner introduces scientific notation to his math class, Jerome asks why they are learning about science during math class. What is the best answer to his question?

Math and other academic subjects work together in the real world.

Mrs. Smithers teaches language arts and is beginning a nonfiction unit. Which of the following should she consider when planning this unit, in order to link instruction with other content areas?

Meet with the social studies and science teachers to see if there is any content she could support with her nonfiction instruction.

Mr. Herrera takes his English class to the library to begin research on the recently-assigned research project. He has sixteen students and divides them into two groups. He takes one group to the computer section of the library to teach them researching techniques using the Internet; the librarian takes the other group to the periodical section to learn how to research using periodicals. After a brief lesson, the groups will switch to learn the other method of research. What is the greatest benefit in dividing the class into two groups?

Mr. Herrera maximizes class time and incorporates the expertise of fellow educators in classroom instruction.

Mr. Kimball is teaching his 7th-grade science class about composting. As Mr. Kimball is describing the process, he realizes that it would be a lot easier to simply take his class to the composting area at a farm a few blocks away from the school. So, he makes an impromptu decision to conduct the lesson at the farm. When the school principal finds out about the excursion, she is visibly upset. Why?

Mr. Kimball didn't get permission from the students' parents and/or guardians.

During adolescence, peers play an important role in psychosocial development and identity shaping. At which stage of adolescence do individuals tend to rely most on peers and least on family?

Mid-adolescence

Ms. Venegas is teaching her students how to use Punnett squares to predict the offspring of a genetic cross. Her students pick up on this topic more quickly than she anticipated, and they finished their activity in one day instead of two. What would be the best course of action for Ms. Venegas to follow?

Move on with the unit and use the extra day for a challenging activity later on in the year.

A middle school teacher, Ms. Andrews, notices that during her lesson many students are not paying attention and a few students are beginning to fall asleep. Which of the following would be the most appropriate action for the teacher to take?

Move to another instructional activity that engages students' interests and reflect on how she can make the lesson more effective.

Mr. Brown notices students chatting with peers in his tenth-grade history class while a group is presenting a final project. What is the best way to respond to this behavior?

Mr. Brown should walk over to the group that is chatting and put a hand on the table of the chatting students.

Mr. Ramirez has overheard several students complain about the current instructional activity. One student said to another, "I am just bored with stuff. I don't understand why it's important and I don't care to know." Mr. Ramirez admits to himself that he has the same feelings toward the material and decides to get through the instructional unit as quickly as possible. Of the following, what is the most likely cause of the students' lack of interest in the material?

Mr. Ramirez does not exhibit an excitement and enthusiasm for the material and his students have adopted his attitude towards the instructional activities.

Teachers are allowed to use some copyrighted material in the classroom using the fair use exceptions. Which of the descriptions below would not be covered under the fair use guideline?

Mr. Swanson makes 100 copies of a novel for students to use in class and at home.

Ms. Abel gave her tenth-grade biology students an exit ticket with five questions and found that 27 out of 30 students did not respond correctly. What is the best next step for Ms. Abel in this situation?

Ms. Abel should revisit this content with the entire class, using a different method to communicate the content to the students.

As Ms. Harrigan is teaching about alternatives to fossil fuels, a student speaks up complaining that alternative sources are not a good idea because they cost more. Which response would best lead the class through a learning experience related to this objection?

Ms. Harrigan says she will incorporate that question into a future lesson, and does it by having a lesson comparing the costs and benefits of alternative energy sources.

Ms. Prabha and Mr. Tsai are teaching their students about the requirements of life. Ms. Prabha plans to give the students 10 unknown specimens and ask them to make observations and then, with a partner, classify them as living or nonliving and give reasons why. Mr. Tsai plans to ask his students to go home and write down the names of 10 living things and 10 non-living things around their house. Which teacher's plan will be more effective?

Ms. Prabha's, because it provides an opportunity for collaborative, student-driven learning. Ms. Prabha can also catch students' misconceptions in real time and gauge student understanding.

Ms. Romero is planning a new collaborative unit in her ninth-grade English class on the Renaissance period, that requires the coordination of history, music, and theatre teachers. She is friends with each of these teachers and they are excited about the cross-curricular project. Which of the following is the best first step for Ms. Romero in planning this unit?

Ms. Romero should meet with school administrators to discuss the parameters of the project and ensure this type of project can be facilitated properly.

Ms. Swick is a technology guru and uses online tools, rubrics, and communication daily in class. She is the campus resource for teachers with new questions. Her students and parents appreciate how easy it is to find resources in and out of class. Which of the following is probably not a method that Ms. Swick uses in her class?

Ms. Swick schedules downtime into her classes so she can implement these resources.

Mr. Stark and Ms. Wendell have very different teaching styles. While supporting one another's instructional methods of choice, there is one thing that they disagree about. Mr. Stark believes that teachers should set low expectations for students, while Ms. Wendell believes in setting high expectations. Who is correct and why?

Ms. Wendell is correct because students tend to meet the expectations that are set for them.

A high school social studies teacher shows a documentary about the role of women during the Civil War. The teacher hands out a worksheet to be completed during the video which asks students several questions about the ideas conveyed in the video, and how the role of women during wartime has changed. This worksheet best reflects the following learning principle:

New ideas and concepts are better reinforced when students are allowed to actively process the new information.

Which of the following curriculum responsibilities would most likely be primarily the responsibility of the classroom teacher?

use of supplemental materials to address course objectives aligned with state standards

After a long day at school, Miss Jackson gets home and opens up Facebook. She is frustrated because a parent has been complaining about her teaching methods and behavior management. Which of the following is acceptable to post as a status update on her social media page?

Nothing. As a teacher, it is unprofessional to post negative comments about education.

Mr. Tomlin is a first-year teacher who wants to improve his classroom management. Which of the following would be the best strategy for Mr. Tomlin to improve his classroom management skills?

Observe experienced teachers in the classroom and discuss classroom management skills when students leave the class.

An elementary school just developed a new science activity center for its first- and second-grade classes. The principal wants to ensure the science center is effective. Which is the best method of evaluation for the principal to use in assessing the center's effectiveness?

Observe the first- and second-graders as they work with the materials and use the resources provided by the center.

Of the following methods, which is not useful in helping students participate in whole-class discussions?

Offer responses when no students speak quickly.

A teacher is scheduled to meet with the parents of a student who is identified as an English Language Learner (ELL). The parents have previously indicated that English is not their first language. Which of the following strategies would be most effective to follow when meeting with the parents?

Offering to invite an interpreter who speaks the parents' native language.

One of the elements that can impact a source's credibility is its publication date. Which of the following best identifies why this element can impact credibility?

Old information may not reflect the current understanding of an issue or idea.

A new student named Elysa joined Mrs. Greer's class halfway through the school year. She recently immigrated to the United States and is an English Language Learner. Mrs. Greer is surprised she did not hear from Elysa's parents when she started in her class. She is even more surprised when they do not come to the school's annual Open House in the spring, even though she made sure to send home the information in their home language. What factor should Mrs. Greer consider when framing her thinking and responding to the situation?

Parent involvement in school is more common in the United States than in many other countries. She should connect the parent to resources that provide a better understanding of the school culture in the United States and common steps to take to become involved.

A seventh-grade math teacher is teaching a lesson that involves creating graphs to chart data from word problems in order to find sums and averages. The teacher has provided direct instruction, and now students are working independently to create graphs. While circulating in the room, the teacher notices that many students are not charting data on their graphs correctly. Which of the following is the best way for the teacher to address this?

Pause the class from independent work and demonstrate how to identify key information and chart data points on the graph.

A tenth-grade English teacher wants her class to use a wide variety of resources and technology in their end-of-year project. She asks each student to use three online sources and two written sources and incorporate a multimedia presentation to show the class. After class, four students who have limited opportunities and technological resources outside of school approach her and express their concern that they may not be able to fulfill all the project requirements because of their limited means. The teacher can best respond and meet the needs of the students using which of the following strategies?

Place students into small groups that consist of students of varying means and abilities while also structuring class time so all students have equal access to the school's equipment. It will be important to let each student know the after-school hours available to access the school resources.

Rather than give a unit test, Mrs. Kirby decides to assign a major project to her students. They are provided a rubric that sets the expectations and guidelines. Students will be given 2 class periods to work on it and the rest must be completed at home. Students will then present their projects in class. What is the main advantage to giving a project rather than a test?

Projects require higher level thinking and can demonstrate greater concept mastery than tests.

Mrs. Cates has assigned a group project that requires students to create multimedia presentations about a historical event that was an effect of the Civil War. The students' presentations contain many graphics and large fonts but lack quality content. Mrs. Cates decides that for future projects she will need to make an adjustment to the project requirements and expectations. Of the following, which would be the most effective strategy to promote quality content among student projects?

Provide a rubric with clear expectations and requirements.

Mr. Estrada's class is learning about genetics. He first has his students observe the traits of their classmates and take note of which traits are more common. Then, the students practice using Punnett squares and observe patterns of inheritance in the offspring. Last, Mr. Estrada asks the students to predict the most likely phenotypes of offspring without drawing a Punnett square. Which of the activities listed above would be the most likely to elicit critical thinking from the students?

Predicting phenotypes of offspring

Mr. Maher has implemented a new behavioral management program. Soon after the program is implemented, a student demonstrates an unacceptable behavior, and Mr. Maher schedules a parent-teacher conference with the student's parents. Which of the following would be the actions that Mr. Maher should take to begin the meeting on a positive note?

Prepare notes about the student's academic achievements to present to the parents and encourage the parents to discuss their observations of the student's academic performance and behavior.

Students in a music theory class have been assigned to use the internet to complete a research project. Which of the following describes how the teacher can best prepare the class to conduct their research?

Provide instruction that guides students through the process of evaluating internet sources for validity and accuracy.

During the intermediate and middle school years, it is recognized that emerging adolescents have a need for independence and opportunities for variety in their classroom instruction and assessment. Which of the following would be the best method to meet these needs when planning lessons?

Provide instructional activities allowing students to make choices from a variety of teacher suggestions concerning their classroom assignments and activities.

A fifth-grade class will be using electronic sources while researching for an upcoming project. The teacher has conducted a lesson reviewing how to determine a source's credibility and practiced previewing a website for relevant source information. Which of the following would best help students ensure their sources are credible while researching on their own?

Provide students with a reference list detailing ways to determine a source's credibility.

Which of the following is the most effective way for a teacher to help a tactile learner understand the consistency of Earth's mantle?

Provide students with modeling clay to touch and squish while the teacher describes the mantle.

Mr. Graham is introducing a lesson about the US Constitution to his students. He periodically calls upon students to assess their knowledge as he lectures. When a student cannot answer a question after an appropriate amount of time, what strategy should Mr. Graham use to best support the student's learning?

Provide the student a brief prompt to help them remember the correct answer.

A school wants to encourage more parents of English language learners (ELLs) to participate in the school's Parent-Teacher Organization (PTO). Which TWO of the following things can the school do to encourage parents of ELLs to get involved in the PTO? Select all answers that apply.

Provide translators at PTO meetings Discuss topics of interest for ELL families at PTO meetings

Passed in 2004, the set of federal regulations known as the Individuals with Disabilities Education Act (IDEA) guarantees students which of the following rights?

Public schools must deliver special education services to students who qualify.

Ms. Simpson is doing a lesson on peer editing and wants to set the room up in a manner to best facilitate the activity. Which of the following should be her focus?

Put desks in small groups so students can pass papers and have discussions on their work.

Which of the following accurately compares qualitative and quantitative assessments?

Quantitative assessments are objectively measured and qualitative assessments measure attitudes, perspective, and observations.

It is a teacher's responsibility to present themselves to their students as a lifelong learner in order to encourage their students to do the same. Which of the following are effective ways for teachers to ensure that they are staying up-to-date on the latest research in education? Select all answers that apply.

Reading recent publications about education technology Attending reputable teaching conferences Becoming active in social media groups made for educators

A middle school parent of a student with disabilities confides in the special education teacher. Her perception is that there is a negative rapport between the English Language Arts teacher and her daughter, and this impacts her daughter's academic and social progress. Which of the following is not one of the responsibilities of the Special Education teacher?

Report the conversation to the Human Resources department.

In Mrs. Deaver's fifth-grade classroom, many students seem to enjoy using a word-processing program for writing. However, there are only four computers in the room. How could the teacher best make use of the limited classroom computer access?

Reserve computers for revisions; facilitate student rotations with a computer station to provide time for all students.

Mr. Burnett's 10th-grade chemistry class had a very low class average on their most recent exam, which focused on the fundamentals of chemical structure. He is worried that because of the students' limited comprehension of this core information, they will struggle with the next unit on chemical bonding. What should he do?

Reteach the core concepts using a varied instructional approach.

Mrs. Sims notices that Sally is having difficulty remembering assignments and turning in her homework. Mrs. Sims contacts Sally's parents for a parent-teacher conference. At the conference, Sally's parents ask how they can help ensure that Sally remembers her homework assignments. Of the following, what is the best recommendation for Mrs. Sims to make?

Review Sally's homework list each night and then help her find a quiet place for her to study.

An 8th-grade class is studying accuracies and inaccuracies in the media, and students have been asked to self-select an article and rate the article according to the validity of the information. A student has found an article on a reputable news site but has noticed that much of the information presented in the article does not align with coverage of the event on other sites. What is an appropriate guideline for the student to follow?

Review coverage of the event from other media sites before making an informed decision on whether the information is accurate or not.

Mr. Luther plans a field trip to the local park for his science class. During the morning, they will observe various animals and plants and record their observations. At lunch, Mr. Luther will gather the class to discuss their observations using predetermined questions. Which of the following instructional activities would most benefit the students and increase the chance for success of the field trip?

Review the questions before allowing the students to begin their observations.

When learning about normal distribution in a bell curve, Mrs. Conway had students create a bell-shaped graph. She wanted to encourage students to be self-motivated in their work, so she offered prizes for the most creative subject matter displayed. Why was this incentive method wrong?

Rewards decrease intrinsic motivation.

The fifth-grade ELA teacher informs the special education teacher that RtI is a means to place students into special education. All of the following are appropriate responses except:

RtI is the first step to the special education referral process.

Mr. Franco has noticed that Sandra, an ELL student in his class, is always doing something with her pen, whether doodling, making a list or taking notes. From this information, Mr. Franco might infer that:

Sandra is a visual learner.

Martin is a fourth-grade student who has Attention Deficit Disorder (ADD) and needs to be in a quiet space while taking tests. Which of the following pieces of legislation is best used to ensure Martin is able to get the accommodation for testing that he requires?

Section 504 of the Rehabilitation Act

Which of the following federal mandates helps remove barriers to education by ensuring a free appropriate public education (FAPE)?

Section 504 of the Rehabilitation Act

Mr. Hampton, a seventh-grade mathematics teacher, receives an email from a parent, Mrs. Anderson. She is complaining about the amount of homework required of her child each night, arguing that the workload is too extensive for any middle school student. Mrs. Anderson requests a parent-teacher conference with Mr. Hampton to discuss the situation. Which of the following strategies is the best option for Mr. Hampton to ensure a productive meeting with Mrs. Anderson?

Seek to understand Mrs. Anderson's concern with the workload and offer the reason for requiring such work.

Mr. Maher wants to create contact with the parents to gain their support for a behavior management program at the beginning of the school year. Which of the following would be the best approach to gaining the support of the parents?

Send a letter home with the students after the first day of school letting the parents know about the behavior program and requesting a parent-teacher conference to discuss the program further.

A kindergarten teacher would like for parents and guardians to feel welcome and involved in the classroom. Which of the following would best communicate these ideas?

Send a survey to parents with a variety of ways they can contribute to the classroom.

Mrs. Jones sends emails to the parents and guardians of her students almost daily with reminders and updates on curriculum and homework. She is frustrated because parents continue to miss deadlines for forms and students will have missing work which is listed in the emails. What can Mrs. Jones do to improve her communication?

Send a weekly email with all important information and only send additional emails if there is a specific concern with an individual child.

A middle school science teacher is planning an intensive lab and would like to have volunteers help. What is the best method to handle sign ups?

Send all parents an invitation to volunteer and take names on a first-come, first-served basis.

Mr. Brown and Mr. Gates decide to split teaching a new social studies program at Yosemite High School. Which of the following is the most important step to take at the beginning of the program to ensure the effectiveness of the program?

Set aside time each week to discuss the next week's activities and the current needs of the students.

Which of the following is an effective way to promote a student's sense of belonging in a cooperative classroom community?

Set aside time for a weekly classroom meeting that addresses issues and focuses on solutions.

Which of the following would be the best strategy to help students work efficiently and effectively while using school resources over an extended amount of time?

Share resources with students and allow them to edit and update the information so all are able to utilize the online tools.

Rachel is more physically developed than her sixth-grade peers in a heterogeneous class of computer literacy. As part of a daily routine, students are given five websites they can choose to explore individually. Although she does not disrupt the class, Rachel never selects a website and often ignores directions, opting instead to write angry messages. Based on this information, what is the best assessment of Rachel's behavior?

She is responding to personal, physical, or social changes and interactions.

Ms. Smith gives a unit exam on transformations of geometric figures. It is a 20-question multiple-choice exam. All but one student gets a 100 percent on the exam. What can be said about her assessment? Select all answers that apply.

She probably should have added some open ended questions to give her students more opportunity to demonstrate mastery. Her students are very good at answering multiple choice questions pertaining to transformations.

Ms. Preston is teaching her sixth-grade students how to create a Works Cited page to be included at the end of their research papers. Jackson, one of her students, asks her if he can use the citation generator found at Scribbr.com. Ms. Preston has never heard of this website. How should Ms. Preston respond?

She should tell him that she'd like to preview the site first to determine its quality.

At the end of a science unit, a teacher needs to assess each student's understanding of the material. Students are given the option of taking a test, completing a project, or writing a paper to demonstrate mastery. What is the greatest benefit of this practice?

an increase in intrinsic motivation

Mr. Baker's school is located in the Rocky Mountains of Colorado. Most of his students have never been to the ocean, nor have they experienced tides. Still, he is supposed to teach his students about tides. In this situation, which of the following activities is likely to be the most helpful when starting the unit on tides?

Show a movie that clearly illustrates the phenomenon of tides.

Mr. House is a first-grade teacher and wants to convey high expectations to a student who has significant academic needs. Which of the following approaches would be the best way to achieve this?

Sit down with the student and work to help set challenging learning goals. Then, follow up and provide assistance to the child to help in achieving the goals.

A school district's budget was cut, and as a result, some funding was lost for the ESL program. Fortunately, several members of the community have expressed an interest in volunteering to help with the ESL program. What two ways would be the most effective use of the volunteers? Select all answers that apply.

Spending time speaking with students in conversational groups Assisting teachers in the classroom with administrative duties that don't require teacher training

Which of the following classroom practices best models Eric Erikson's behavioral theory?

Students are given differentiated assignments that allow them to succeed often.

Which of the following situations is an example of students using their First Amendment rights in the classroom?

Students debate the use of stem cells in medicine and research.

Ms. Hill often uses flexible grouping with her second-grade class. For which of the following reasons might Ms. Hill choose to group her students homogeneously for a task?

Students differ in their preferred method of accessing content (e.g. reading, listening, watching videos, etc.).

Ms. Lavine plans a lesson with the following learning outcome: "Students will write an introductory paragraph with an engaging opening statement, two supporting sentences, and a topic sentence." After teaching and modeling the creation of an introductory paragraph, Ms. Lavine gives the students some time to create their own introductory paragraphs. Once completed, she pairs the students for a peer conference on one another's writing. She explains that it's important for students to focus the feedback on the lesson objective and instructs them to clarify what parts of the writing are strong and explain why other parts might need to be strengthened. The students then form pairs and proceed to critique each other's work. This is the first time that Ms. Lavine has had her students engage in peer conferencing. She walks around the room to observe, but does not see the desired results. What should Ms. Lavine consider before using this strategy again?

Students often need to be exposed to a strategy over time before seeing the desired results.

Which of the following learning goals is most appropriate when teaching an eighth-grade class about cultural contributions to mathematics?

Students will be able to explain how various cultures have contributed to the development of mathematics throughout history.

Which of the following best identifies an observable and measurable instructional objective for a unit of study on poetry?

Students will compare and contrast haikus and cinquain poems using a Venn diagram.

Mr. Sanders is preparing a lesson for the following TEKS History standard: The student understands how individuals, events, and issues shaped the history of Texas during the 20th and early 21st centuries. The student is expected to: (A) explain the political, economic, and social impact of the oil industry on the industrialization of Texas. Which of the following would be an example of a learning goal to support this TEKS standard?

Students will differentiate between the Texas economy before and after the rise of the oil industry.

Which of the following describes a significant problem that will result from a teacher failing to establish a classroom routine at the start of each class?

Students will have difficulty focusing on learning at the beginning of class.

Which of the following is the best rationale for using formative assessment?

Students will show what skills they have mastered and what skills still need to be practiced.

Ms. Kingly is rather unsure about the cause of the Coriolis effect, so she decides to show a movie about it to her class instead of trying to teach it to them directly. What else should Ms. Kingly do in order to prepare for the lesson?

Study up on the Coriolis effect so she can confidently answer student questions and guide further exploration.

Students in an eighth-grade math class represent a wide range of skill acquisition and cognitive development, from concrete operational to formal operational thought. What is the teacher's best strategy for planning instruction of a geometry unit for this degree of cognitive variation among students?

Take students on campus to identify concrete examples and construct models of various geometric shapes, in order to illustrate abstract content.

Which of the following represents the stages of cognitive development you would expect for most high school students?

concrete operational and formal operational

At which of the following phases do children begin to think logically?

concrete operations

A new teacher is giving her first social studies test and needs to modify it for her students with an IEP. She is unsure of how to change the questions so she just spaces them out and removes one answer choice. Overall, the students who take this test do poorly. What should the special education teacher do?

Talk with the new teacher and offer to modify the test together next time so that it is aligned with the IEP goals.

A high school teacher is looking to make changes in the classroom to promote a more inclusive environment that will increase social and emotional learning (SEL). Which of the following changes is likely to have the most positive impact?

Talking to students about personal interests and sharing similar experiences.

Which of the following examples would best help students to improve their skills of metacognition?

Teach learning strategies and have students pinpoint which work best with new assignments.

Mr. Barrios is teaching a unit on multiplication to his fifth-grade class. On the very first day he gives an exit slip with the following problem on it: 123.456 \times 789 =123.456×789= _______ Every single student gets the question correct. How should he adjust his teaching?

Teach more advanced multiplication content to challenge his students.

A second-grade teacher has noticed that her students are struggling to express their emotions in an appropriate way. They get frustrated or mad at a friend and pout until another situation distracts them. What would be the best method to address this issue?

Teach students a script to use when they've been upset. For example: Student 1 says, "It hurt my feelings when ____." Then, Student 2 responds, "I'm sorry. Please forgive me for ___."

To facilitate the development of higher-order thinking skills, a teacher could: Select all answers that apply.

Teach students how to label the type of question being asked and use that information to assist in formulating a response. Encourage students to post questions on a board to be reviewed at a later time, when class time doesn't allow for discussion. Praise students for following a set method for accomplishing a task, in addition to arriving at the correct answer.

A kindergarten teacher observes the following conversation between two students at the kitchen center. Marcus: (Loudly) Hey! I was playing with that! Jose: You have some food. I want this pot! Marcus: I was here first! You're stealing. I'm telling! Jose: (Yelling) You're mean and I'm not your friend anymore! Which of the following steps should the teacher take to alleviate this situation?

Teach students how to talk calmly and agree on what to play together

Mr. Ivin's algebra class is learning the pythagorean theorem. He has two students that are auditory learners and are struggling with the concept. What activity would best help them learn the theorem?

Teach the class a short song that has the key components of the pythagorean theorem.

Which of the following statements is the best example of something positively extrinsic?

Teacher: If you complete the bonus questions, you can receive extra credit.

Which of the following is not an expectation of teachers to develop open communication with parents/guardians?

Teachers should email parents weekly with an update about their child's progress.

A teacher wants to accurately assess students who have diverse backgrounds, strengths, and needs. Which of the following could be used both formally and informally to assess their comprehension of the classroom material? Select all answers that apply.

Tests performance Classroom observations Class discussions

A new student began at Spring Memorial Middle School a month ago and has qualified for special education services for a specific learning disability in reading and writing. She recently moved from Mexico and her special education testing was done in Spanish. During the initial ARD committee meeting, they address accommodations for the classroom and STAAR test. What other test should be included in the IEP documents for accommodations?

Texas English Language Proficiency Assessment System (TELPAS)

In discussing the state assessment, State of Texas Assessment of Academic Readiness (STAAR), the parent wants to understand how it would be supported in the IEP. Which response should the ARD committee chair offer the parent?

The IEP objectives closely align with the STAAR assessment for all students.

How should the Texas Essential Knowledge and Skills (TEKS) be reflected in a student's Individualized Education Program (IEP)?

The IEP objectives should try to align as closely as possible with the appropriate TEKS.

Which of the following website titles represents a credible source?

The National Science Journal

Mrs. Archer is planning the first unit for her upcoming school year. Which of the following best describes the role of the Texas Essential Knowledge and Skills (TEKS) in her planning process?

The TEKS should be used as a guide to determine instructional goals.

What role do teachers and parents/guardians play in the academic success of elementary students?

The adults set high expectations, which help students succeed.

Children develop language skills as they grow. Which of the following represents age appropriate language skills for a child when is seven years old?

The child expresses opinions and holds conversations.

In which of the following situations should a diagnostic test for evaluating student(s) be used?

The class is about to begin instruction in a new unit.

A middle school science teacher asks students to complete a concept map whenever they are reading about a new topic in the textbook. Which of the following describes the primary reason for providing students with a concept map?

The concept map helps students make connections between unfamiliar topics.

When deciding if a source is credible for a research project, which of the following is important for the student to consider? Select all answers that apply.

The credentials of the author The author's purpose The publication date

Ms. Lavine plans a lesson with the following learning outcome: "Students will write an introductory paragraph with an engaging opening statement, two supporting sentences, and a topic sentence." After teaching and modeling the creation of an introductory paragraph, Ms. Lavine gives the students some time to create their own introductory paragraphs. Once completed, she pairs the students for a peer conference on one another's writing. She explains that it's important for students to focus the feedback on the lesson objective and instructs them to clarify what parts of the writing are strong and explain why other parts might need to be strengthened. The students then form pairs and proceed to critique each other's work. Which two of the following components of effective feedback should Ms. Lavine emphasize to help students in their peer conferences? Select all answers that apply.

The feedback should be specific. The feedback should be academically focused.

Of the following, which is the most important feature of an instructional assessment?

The learning objectives are assessed and do not vary significantly from the material the students were taught.

A teacher wants to introduce a new concept to the students. During the student's first experience in working with the new concept, which of the following should be the main focus of the teacher?

The main focus should be analyzing the students' understanding of the concept and providing constructive feedback.

A middle-school with a high percentage of ELLs notices that only a small percentage of the ELL parents have been participating in school meetings. What is the most likely cause of this?

The meetings are held in English, and there are no translators available.

A department meeting with fifth-grade and sixth-grade language arts teachers is held to discuss a lack of student ability to recognize and understand the use of root words and affixes in the vocabulary-building process. A new teacher states that at the elementary school, a big emphasis is made to align different roots and affixes by each grade level so that a logical progression can be made from one grade to the next. The intermediate teachers decide to meet with the teachers at the elementary feeder school to see what approaches have been used successfully in the fourth grade to reinforce these concepts. In addition, the teachers thought it might be beneficial to meet with the seventh-grade LA teachers to determine if there is any pattern in student reading or writing skills that would indicate a lack of a quality foundation that they could be reinforcing at the intermediate level. Which of the following would be the best way to affect this plan?

The new teacher is asked by the department head to help present the idea to their principal and solicit his help by discussing it with the middle school principal.

A few students are struggling with the current learning objective. How should this observation impact the pacing of lessons for the class in this unit?

The pacing should remain the same because most students have been successful.

A parent disagrees with a school's decision regarding a disciplinary violation of a student which has resulted in a 10-day suspension. Which of the following correctly describes a parental right?

The parent has the right to request to examine the student's educational records.

When beginning a parent-teacher conference, who should start the conversation?

The person who requested the conference should, so they can share all of their concerns and focus the meeting.

Which of the following is the best reason a teacher should mindfully include science practices and interdisciplinary connections when planning a science lesson?

The practices and connections will be useful in the students' future studies and careers.

Mrs. Gruntle is planning a health unit on substance abuse. Before beginning the unit, Mrs. Gruntle prepares a pretest to give the students. Which of the following is the most important advantage of the pretest?

The pretest will help Mrs. Gruntle recognize misconceptions students have about substance abuse

Which of the following is an accurate statement concerning the guidelines for record keeping?

The rules concerning records can vary by state, but accuracy and confidentiality are universal.

An eighth-grade student is having difficulty writing down homework and staying organized when gathering materials to transition to the next class period. Which of the following strategies would be most effective in helping the student to become better organized and implement these strategies when preparing to transition between classes?

The student uses a self-monitoring chart that lists steps to take to record homework and organize materials before leaving the classroom.

A student with disabilities is enrolling in a new school. In order to prepare for this new student, the school should understand the legal rights of the student and the responsibility of the school to:

deliver instruction to the student in the least restrictive environment.

A language arts class has been working on correctly constructing and identifying the components of simple sentences. At the beginning of the following class period, the students complete a warm-up designed to assess their knowledge of simple sentences. After turning in their warm up, the teacher immediately transitions into instruction on compound and complex sentences. The teacher plans to provide feedback on all three skills after grading all the assessments over the weekend. Which of the following best describes the problem with this plan?

The students and the teacher need to know the results of the simple sentences assessment before moving on to more complex sentence structure.

A new freshmen teacher plans to place her class in instructional groups by their achievement scores of last year. She announces that each group will have different assignments based on their level of past performance. Which of the following would best predict her students' attitudes and behavior using this practice?

The students would feel that the teacher does not have very high learning expectations of all of the groups and would feel marginalized.

Mr. Banner is a teacher that utilizes a variety of instructional activities, such as lectures, group projects, videos, and research projects in his classroom. Which of the following describes the greatest benefit of using a variety of instructional activities in a classroom?

The teacher addresses the different learning styles of the students.

Which of the following best explains the reason for using homogeneous grouping in the classroom?

The teacher can provide direct instruction to small groups that meet their current instructional needs.

How can a teacher use informal assessment to guide instruction?

The teacher can target students' specific problem areas and adapt instruction to address those areas.

In which of the following situations would it be most important for a high school English teacher to consult with a school administrator or school counselor before proceeding with a lesson plan?

The teacher is planning to have students read a book that includes traumatic events, and several students in the class were personally impacted by this type of trauma

Which of the following is not a useful, non-verbal method of communication to encourage students to refocus their attention on their classroom work?

The teacher leaves the classroom to go into his private office when students are talking out of turn.

Which of the following is the best method for a teacher to improve the divergent thinking skills of students?

The teacher poses a problem and prompts students to explore potential solutions using a concept map.

A third-grade student informs the teacher that he is constantly being called inappropriate names by other students in the classroom. He feels that he does not belong to any group of students in the class. The teacher monitors the behaviors of the students and agrees that the student is frequently excluded from activities. How should the teacher respond to this situation?

The teacher should design collaborative group and team building activities to promote a learning environment inclusion for all students so they feel a sense of belonging and safety.

A new teacher will be working at a school that has many types of professionals on the staff, including paraprofessionals, speech/language therapists, and school psychologists. Due to the nature of the student population, many of these professionals will be in and out of classrooms on a regular basis. What is the best way for the new teacher to form relationships with these professionals?

The teacher should meet with professionals who will regularly be in his/her classroom to learn more about their roles and how to collaborate effectively to support student learning.

Mrs. Johnson has previously taught seventh-grade science and now has been assigned to teach science in the eighth grade. She introduces the topic of pollution by carefully crafting the following lesson using a DVD. Students are told that the DVD's objective is to cover critical information concerning water and air pollution in the US. Then, they are directed to work in groups at their tables and write five questions per group that they expect to be answered while viewing the DVD. She sets the timer for seven minutes to work. When the timer goes off, each group states two questions they wrote, trying not to duplicate responses. The teacher then explains that they will actively watch and listen to see if their questions get answered. The portion of the DVD they view takes 14 minutes. She then asks them to individually reflect on what they thought was the most interesting thing they viewed in the DVD and if their questions were answered. After a full minute of wait time, a teacher-facilitated discussion takes place concerning the students' interests and the answers to their questions. Mrs. Johnson then requests that the class members personally write in their science journals, rating the DVD on a scale of one to five, explaining why they rated the video with this score and their reflections about what they learned by its viewing. Which of the following statements would be the most accurate evaluation of this entire lesson?

The teacher used a variety of appropriate motivational, individual, and group activities and allocated adequate time for self-assessment and closure.

Which of the following strategies is most likely to improve a teacher's communication with parents/guardians?

The teacher uses common terms that all parents can understand rather than relying on pedagogical terms.

Mr. Hamilton places a high respect on critical-thinking skills in his class. He wants to encourage his students to think through their answers with more care and consideration, and to increase student participation in content discussions. Mr. Hamilton decides to increase the amount of time he waits after asking a question, before providing more information and/or answering the question himself. When this strategy is used effectively, Mr. Hamilton will use an increased wait time: Select all answers that apply.

When asking the students complex cognitive questions When asking a question to the entire class and waiting for a volunteer When discussing class content one on one with a student

Prior to a first-grade teacher reading a story about the zoo to his class, he tells his students that he wants them to learn which animals live at the zoo because they will be going on a field trip to the local zoo next week. By telling the students the objective of the reading material, the teacher best demonstrates knowledge of which teaching principle?

When students are aware of the focus and expectations of an activity, their comprehension of the material increases.

Which of the following is the greatest educational benefit of using collaborative learning groups and active learning activities in the classroom?

These methods address a variety of learning styles and allow students to take ownership of their learning.

Research has shown that ELL students benefit from formative assessments. All of the following are benefits that formative assessments provide to ELL students except:

They provide students with a grade that can be compared to peers.

Before introducing a new concept, a middle school teacher always uses a K-W-L activity. Which of the following is the primary reason for the teacher to use this activity at the start of each lesson?

This activity provides a baseline for students' current knowledge so that instruction can be planned accordingly.

What is the purpose of a scope and sequence document?

This document, usually provided by the district, gives a high level view of information and skills that students are expected to learn over the course of the year in a reasonable and useful order.

While reviewing the syllabus at the beginning of the school year, a science teacher informs students that the course will have weighted grades in which 40% of the grade will be made up of assessment items, and 60% of the grade will be made up of classwork, homework, and participation grades. What is the advantage of this assessment practice?

This ensures the student will be graded fairly and that the teacher will have information about students who are falling behind.

At the start of each lesson, Mr. Goodman asks his students to raise their hands and state something important that they learned in the previous lesson. He writes their responses on the board and then refers back to them during the class that day. What is the main purpose of doing this?

This is a way to help students retain new knowledge.

A teacher wants to improve student interest in social studies. During the next unit, which is historical, the teacher plans to read personal narratives about key people that describes parts of their childhood and personal triumphs while achieving success. How does this practice increase student engagement?

This practice makes the content concrete and relevant to students' lives.

An 8th-grade history teacher is teaching his students about past presidents. As part of the unit, he plans to have a mock election, in which each student acts as a former president and presents a persuasive campaign speech. The following steps will be performed over a week. Step 1: Students will be shown a video clip of two presidential nominees giving a campaign speech. Students will be asked to share their opinions on the candidates. The teacher will reveal who the candidates are and who won the election. Step 2: The teacher will present the elements of a persuasive speech. The students will watch another video and identify the elements within the campaign speeches. Step 3: Students will be asked to name some former presidents and their accomplishments. Students will complete a graphic organizer matching each president to his major accomplishment. Step 4: Students will be assigned a president to research and act as when giving a campaign speech. The teacher will randomly select sets of two presidents to compete against one another in a modern-day mock election. Step 5: Students will research their president and prepare a 2-3 minute campaign speech, addressing the issues of today, for a mock election. Additionally, students will prepare a campaign slogan and poster. Step 6: Students will give their speeches and classmates will vote for a winner. What should the teacher provide students in order to ensure the successful completion of the task?

a prepared research outline or rubric

A teacher is planning an introductory lesson for a new unit on potential and kinetic energy. The teacher wants to increase interest and motivate student learning. The teacher should include which of the following in the first lesson?

a real-world example that many students have encountered

According to B.F. Skinner, which of the following is an example of a positive reinforcement?

a reward is offered; a desirable behavior increases

Mr. Dickens is an advanced placement English teacher. To help students prepare for the AP English exams, he provides frequent essay assignments on the literature read by the class. He outlines the grading requirements for the essays in terms of grammar, punctuation, usage, logic, the organization of ideas, etc. Which of the following would be most appropriate to provide to the students to convey Mr. Dickens' grading requirements?

a rubric

True or False: The transition plan for a high school student is reviewed and updated every year during the annual Admission, Review, and Dismissal (ARD) committee meeting.

True, this is a requirement of the IDEA and the Texas Education Code

Mr. Harris wants to create a writing assignment that allows students to write with an authentic and meaningful purpose. Which one of the following assignments would best accomplish Mr. Harris' goal?

Use Word to type a letter to an elected official about a student-chosen need in the community. Print and send the letters.

A teacher is disappointed by her students' exam results and decides that it's necessary to re-teach some of the concepts, as the students need a strong grasp on these concepts before continuing on to the next unit. To effectively reteach, which of the following should the teacher do? Select all answers that apply.

Use a different approach from the one used in the previous instruction Group the students who have similar needs

Mrs. Sims wants to teach her first-grade science class how parts work together to allow systems to work, and without all the parts, systems may not work properly. Which of the following instructional strategies is the best way to convey this concept?

Use a system the students are familiar with as an example, such as demonstrating that a computer does not work without a power cord.

To avoid plagiarism, a student should give credit whenever he/she does which of the following? Select all answers that apply.

Use another person's idea, opinion, or theory Paraphrase a person's spoken or written words

Which of the following activities would be most effective for a tactile learner to learn about the phases of mitosis?

Use color-coded pieces of yarn to represent the chromosomes. The yarn acts as a manipulative the student can use to represent the phases of mitosis.

Which is the best way to help elementary students learn the scientific method?

Use ideas from the scientific method, with explicit instruction, in hands-on investigations throughout the year.

Mr. Appleton is considering providing some material for his students from the internet. Which of the following is Mr. Appleton legally permitted to do under fair use guidelines?

Use pictures and information from various websites in a class presentation.

One of Ms. Landers' 4th-grade students is high achieving and enthusiastic about school. She becomes frustrated when not called on to answer questions and is known to take over when working collaboratively in groups. How can Ms. Landers best support this student along with the rest of the class?

Use role-playing games to model positive small and whole group interactions while also finding opportunities for this student to take an active leadership role in the class.

A fifth grade teacher uses science labs to reinforce concepts. The teacher would like to have students use self-assessment skills as a part of the process of evaluating their lab reports. Which of the following is the most effective method of self-assessment?

Use scaffolding to guide students in creating the evaluation criteria, then have students use those standards in assessing their own work.

Mr. Ray typically begins each day with direct-teaching instruction where he uses the whiteboard and lecture notes to teach the students. Many times, he has turned around from writing on the whiteboard to find students distracting each other. Of the following, what would be the most effective technique to maintain the students' interest?

Use short multimedia presentations such as PowerPoint to deliver content that is accompanied by pictures or short videos.

Which of the following strategies would be most effective if a teacher were attempting to increase parents' involvement with their children's education?

Using a variety of communication methods and being flexible when scheduling conferences.

Mr. Daniels has assigned his class a group project and wants to ensure the students understand the project as they complete it. Which of the following would be the best strategy to monitor student understanding during the project?

Walk from group to group while observing the students.

Students in Mr. Miller's class are given stickers for high grades and positive behavior, while they lose recess time and must redo poor work or practice better behavior if needed. This is an example of which theorist's ideas?

Watson

Which of the following correctly explains the primary reason students should avoid doing research on websites ending with ".com?"

Websites that end in ".com" are commercial entities that are potentially biased because their primary purpose is to make money, not to inform.

Which question below would help fourth-grade students to use metacognition when completing reading comprehension questions on a standardized practice test?

Which learning strategies have you used for this question?

After each lesson, a teacher reflects on the lesson in order to consider formative assessment needs. Which of the following questions would be best for the teacher to consider?

Which objectives from today's lesson do my students still need to master?

Mr. Blanson is planning a unit on land formations. Which of the following would serve as an effective introduction activity to activate prior knowledge?

With students in pairs, provide a list of land formations and ask the students to sketch a picture of each from memory.

After several lessons in Newton's laws, Mr. Obangi presented his class with the first of several difficult questions based on misconceptions about Newton's laws. The students used clickers to register their answers, but only about 75% of the students selected the correct answer. Which of the following is the best follow up for Mr. Obangi?

Without telling the correct answer, Mr. Obangi should have students discuss their reasoning with one or two neighbors, after which students click in new answers to the question.

The third-grade teachers at Treasure Forest Elementary School are planning a unit on Thomas Jefferson. They want to challenge their students using higher levels of Bloom's Taxonomy, so they are brainstorming ideas to replace their current activities. Which of these requires the highest level of thinking?

Write an essay about the best character traits of Jefferson.

Mr. Mahoney teaches science in a public school and is working over the summer to plan experiments for the upcoming school year. What guiding document from the school or district will he need before beginning his planning process?

a scope and sequence

Which of the following represents a criterion-referenced score?

a score that indicates how well a student understands the specific content on a project

An 8th-grade history teacher is teaching his students about past presidents. As part of the unit, he plans to have a mock election, in which each student acts as a former president and presents a persuasive campaign speech. The following steps will be performed over a week. Step 1: Students will be shown a video clip of two presidential nominees giving a campaign speech. Students will be asked to share their opinions on the candidates. The teacher will reveal who the candidates are and who won the election. Step 2: The teacher will present the elements of a persuasive speech. The students will watch another video and identify the elements within the campaign speeches. Step 3: Students will be asked to name some former presidents and their accomplishments. Students will complete a graphic organizer matching each president to his major accomplishment. Step 4: Students will be assigned a president to research and act as when giving a campaign speech. The teacher will randomly select sets of two presidents to compete against one another in a modern-day mock election. Step 5: Students will research their president and prepare a 2-3 minute campaign speech, addressing the issues of today, for a mock election. Additionally, students will prepare a campaign slogan and poster. Step 6: Students will give their speeches and classmates will vote for a winner. Which two of the following standards would fit well with this unit? Select all answers that apply.

Write persuasive texts to influence the attitudes or actions of a specific audience Produce clear and coherent writing in which the development, organization, and style are appropriate to task, purpose, and audience

Mr. Frango is teaching his ELL students how to write persuasive essays, but they are still struggling to correctly use subordinate conjunctions to express their ideas. Is it necessary for him to reteach his students?

Yes, if they are high-intermediate students and understand the concept at a minimal level.

Which of the following would NOT be considered a reliable research source?

a blog

In the middle of a lesson on plant systems, a teacher asks students to raise their hand if a statement she says about roots is true. She reads four statements and notes how accurate the class is for each statement. This exercise is an example of:

a formative assessment.

Mr. Herman is a new principal at Hardy Elementary school. During his training period, he learns that Hardy Elementary school has a 25% new teacher attrition rate. Mr. Herman, shocked that this number is so high, vows to provide more support and encouragement to new staff. To do this, he plans to: Create a detailed teacher orientation program to assimilate new teachers to the school and community culture Develop a high-quality teacher mentoring program to support, guide and encourage new teachers Dedicate more resources to collecting teacher feedback at consistent points throughout the year Develop school committees in which teachers can collaborate with one another Seek ways to be more progressive and provide ongoing support As part of his committee development process, Mr. Herman would like to create committees from different content areas within the same grade level to share cross-curricular learning ideas. What type of committee should he form?

a horizontal team

Mr. Everett teaches a class of first-grade students with very mixed ability levels. Based on the flexible grouping model, which of the following would be the most appropriate way to group students?

a mix of homogeneous and heterogeneous groups of varied sizes

Which of the following is a norm-referenced test?

a multiple-choice test that evaluates basic reading skills

A new middle school science teacher is planning lessons and units for the semester. Which of the following resources would be most useful for developing these types of plans?

a pacing guide for the district that aligns with state standards

A middle school teacher wants to assess the proficiency level of her students with a specific word-processing software. Which of the following assessment procedures would be most appropriate for this purpose?

a performance assessment in the word-processing software

A teacher is teaching an early elementary class that all animals grow and develop over time. Which of the following supports would best enhance a visual learner's ability to explain the changes during the process of a caterpillar becoming a butterfly?

a pictorial diagram of using labeled arrows to indicate the changes

A teacher is teaching a class that all of our food comes from the sun through a series of energy transfers. Which of the following supports would best enhance a visual learner's ability to explain the energy transfers that occur during this process?

a pictorial diagram using arrows to indicate the series of energy transfers

Last year, a high school hired a new programming director, who introduced many different programs to students, both before and after school hours. At the end of the year, the teachers at the school noted that their ELL students seemed to have higher self-esteem than in past years. Which of the following programs might have caused this effect?

a series of multicultural events that introduced students to different cultures and backgrounds through food, music, and games

Ms. McManus has purposefully set a climate of high expectations in her classroom. Which of the following are NOT likely to be seen in her class?

a sign that says "good students are A students"

A test given at the end of a unit on evolution is an example of:

a summative assessment.

Which of the following types of learners would benefit most from the activity described below? A history class is learning about US presidents. The teacher asks each student to pick a president that they want to learn more about, and the class has a day in which the students dress up as the president and provide information about the goals they achieved during their term in office.

a tactile learner

During the first week of school, which of the following is the best assessment a new eighth-grade science teacher can use to determine students' knowledge and understanding retained over the summer?

a teacher-made test on vocabulary and concepts that will be covered during the first grading period

Which of the following scenarios would fall under Maslow's level of belonging and love needs?

a teenager establishing his first relationship with a girlfriend

A science teacher would like to determine students' understanding at the end of a unit. Which of the following activities would be best suited for this purpose?

a test featuring a variety of questioning methods

Mrs. Hogan is wrapping up a unit on homeostasis. She would like to assess student knowledge before she moves on. Which of the following would be the best summative assessment?

a test over homeostasis and the processes involved

A 6th-grade history teacher is reading a text about a Pilgrim's journey aboard the Mayflower. After she completes the reading, the students will write journal entries pretending as if they are passengers aboard the same ship. As she reads the text, the teacher adds some thoughts, saying: "Wow, I can't believe the conditions on the ship were so terrible. The immigrants must have been so brave. I would feel scared, but also excited imagining what the new world might be like. I'm going to write these thoughts in my journal." What strategy is the teacher using to model for her students?

a think-aloud

Which of the following behaviors is not common for a seven-year-old student?

able to write paragraphs

The math teachers on a campus are trying a new method of teaching variables. As they implement the new method, each teacher collects data to determine the effectiveness. What type of research methodology are the teachers using?

action research

Mr. Popov's students are building circuits. The ends of the conducting wires have alligator clips that students use when making connections. Mr. Popov notices that several students are connecting the alligator clips to the plastic that coats each of the conducting wires instead of connecting the metal clips to metal. What did Mr. Popov specifically forget to include in his lesson plans?

activating students' prior knowledge about conductors and insulators

Mrs. Fields is creating a lesson plan for her third-grade inclusion class. Which of the following should Mrs. Fields include in the lesson plan to ensure that all students receive quality instruction?

additional time at the end of each instructional activity to reteach and address the needs-based instruction modifications where necessary

A science teacher uses labs to reinforce concepts taught in class. Students must read the lab as they participate, then turn in a written conclusion that has at least one illustration and one graph. Then, they will receive points for a class discussion about data at the end. What type of learner is the teacher targeting?

all of the above

The Texas education system is made up of primarily independent school districts. This means that each district maintains their own rules and governing board, while also:

all of the above

The purpose of a learning log is:

all of the above

When instructing students to look through primary and secondary sources, the most important reason for offering as wide a variety of research media as possible is to:

allow each student to use the medium that best suits their learning style.

One method teachers can implement to increase self-motivation is:

allowing choice for some assignments.

Ms. Davidson creates a very detailed syllabus at the start of the school year for her 8th-grade science class. Every week she introduces a new concept, and once a week, her students conduct an experiment related to the topic in a science lab. She has been following the same curriculum and using the same materials for years. Recently, she has felt that her students aren't as engaged as she would like. Which of the following activities is most likely to increase student motivation?

allowing students to choose the theme and methods to try in their experiments

In order to develop relationships with families, the teacher invites families to attend open house at the beginning of the school year. At this event, the teacher should provide:

an email and phone number for contacting the teacher.

Which of the following is an example of a formative assessment?

an exit ticket

Use the information below to answer the questions that follow. The information presents a hypothetical classroom scenario, and the questions require that you make the best decision, based on the presented information. Ms. Wright is a 6th-grade math teacher who likes to design her own task-based lessons to go along with the school curriculum. For a unit on statistics, she designs a project in which students create statistical questions, survey others, display and analyze the results and report their interpretations and conclusions in a presentation to the class. After each step, Ms. Wright has the students turn in their work so that she can identify any errors and provide constructive feedback. When the students present their conclusions to the class, Ms. Wright is impressed by her students' presentations and findings but worried because the task has taken two days longer than it should have. Now, she feels pressure to cut down the content in the next unit to get back on schedule. Ms. Wright tells her mentor teacher about her concern, and together they list the specific problems that were causing her delay: 1) Too much time was spent collecting student work and returning it to students. 2) A lot of time was spent commenting on student work and providing feedback. 3) Students often misplaced parts of their assignment, as it was constantly being transferred back and forth from student to teacher. Ms. Wright's decision to allow her students to create their own statistical questions is likely to result in:

an increase in the students' motivation.

Which of the following interactive online tools would best assist a teacher in trying to gauge the understanding of her class in real-time?

an interactive presentation tool in which students can respond on a device to questions posed by the teacher within the presentation

Which of the following describes a credible source?

an online database with peer-reviewed articles

After her students have learned the names and descriptions of various volcanoes, Ms. Garcia asks her class to read some scientific research on volcanoes followed by a discussion on why we study volcanoes. By extending her lesson this way, Ms. Garcia most likely intended to move her students beyond the knowledge level of Bloom's taxonomy to:

analysis and synthesis.

A teacher asks her students to compare and contrast a pair of thesis statements. According to Bloom's Taxonomy, what level best aligns with this task?

analyze

A fifth-grade science teacher has students read several articles about renewable and non-renewable resources. After reading, the students create a Venn diagram to compare and contrast the energy sources. This follow-up activity is likely to be most effective in helping students:

analyze information from the articles.

Which of the following social studies activities would require higher-order thinking skills?

analyzing the checks and balances system in the three branches of government

The Texas Education Code (TEC) sets legal standards for all schools in the state of Texas. What schools are required to abide by these laws?

any public school that receives taxpayer funds

A first-grade teacher schedules time every day to read aloud to her class. She wants to promote an understanding of books and a love of reading. In selecting stories to read, the teacher should focus on choosing books that:

are engaging and enjoyable for the students.

A second-grade teacher uses a portion of each day to have students read silently by themselves while she walks around the classroom assessing their reading abilities. The goal of the silent reading is to promote each student's appreciation of and interest in reading. When selecting books for the students to read, it is most important for the teacher to choose books that:

are engaging and interesting to the students.

A seventh-grade science teacher regularly quizzes students during class time using dry erase boards and markers. After the teacher provides direct instruction on a new concept, students take out their boards and markers to respond to a brief series of oral questions. Students have 1-2 minutes to record their responses and all hold up their boards at the same time to review answers. During a unit on cells, the teacher quizzed students on several important cell parts. Below are two students' responses to a question that asked them to describe the function of the nucleus of a cell. Student 1: The nucleus is like the "command center" or "brain" of a cell. Student 2: The nucleus contains very important information about the genome. Which of the following describes the best way to use students' responses on the dry erase boards?

as an informal assessment to find out if students understand the material

Mr. Johnson always designs his lessons with some flexibility. He likes to give his students the ability to choose between a wide range of options when completing assignments. He does this because he knows that the average middle school student:

has an increasing need for autonomy.

The Dalton School District has access to a short exciting, personal, eye-witness story about a tornado that struck across their state and killed 36 people. The author describes how she felt during the approach, destruction, and aftermath of the tornado. Of the following choices, which is the best place in the lesson plan for the science teacher to read the story to the class?

as an introduction to the unit

On the first day of school, a teacher wants her students to feel welcome and know that they are important members of the classroom. Which of the following methods is best to greet students when she first meets them?

ask students how they would like to be greeted with a visual menu by the door

Which would be the recommended sequence to follow when asking questions in the classroom?

ask the question, wait 5-10 seconds, and then call on a specific student to answer

A second-grade teacher would like to help her students develop their interpersonal skills. Which of the following activities would be best suited to meet the teacher's goal?

assign a project in which each student has a specific role to fulfill for the group

A second-grade teacher would like to form stronger connections between school and home. Which of the following approaches should the teacher take to learn more about her students' lives outside of the classroom?

assign a student/parent survey asking about academic history and study habits

Read the information below and complete the questions that follow. Every week Mr. Brown devotes an entire class period to small group activities. He separates the students into heterogeneous groups, which change each week so that students have the opportunity to work with every classmate at least once. At the end of the month, he reflects on the positive and negatives of group work and creates the following list: Positives Negatives Exposure to different perspectives Improved ability to cooperate with others Enhanced social skills and ability to interact with others More student-centered approach Unequal participation - Some students are passive participants in the group. Can be time-consuming Occasional conflict among group members During group work, Mr. Brown notices that some students always take control within the groups and dominate the task, while other students are more passive participants. What can Mr. Brown do to reduce this problem?

assign each member of the group a specific task to complete

Which of the following activities for studying cell organelles would best serve a kinesthetic learner?

assigning each student an organelle and acting out a play about them

A community member is interested in volunteering with a school's ESL program. Which of the following tasks would be most appropriate for him to do?

assisting with the tutoring of students who need speaking and writing practice

A new teacher has struggled with classroom management throughout the year. Which of the following opportunities would be most beneficial to improve her skills in this area?

attending a local workshop on building relationships with students

Henri is an ELL student from France. Henri's teacher has observed that he likes to read out loud to himself and he often repeats what she says in his own voice. He isn't afraid to speak up in class and always performs well on listening tasks. According to these details, Henri's teacher can infer that he is a(n):

auditory learner.

An elementary teacher wants to be sure the lessons offered are presented in a logical manner that enhances student understanding. The teacher sets learning goals and then plans the unit around these goals. This method is often referred to as:

backwards planning.

A teacher is leading a discussion on ancient China. A student asks a question that the teacher does not know the answer to. In order to model intellectual curiosity and the pursuit of new knowledge, the teacher should do which of the following?

be honest about the lack of knowledge and then lead the class in researching and discussing the answer

Choose the answer below that best completes the idea. Within a classroom, a teacher is planning to use ability-grouping for math instruction. To minimize potential negative effects, the teacher should:

be willing to rearrange groups based on their specific learning needs.

A student is having difficulty using appropriate language and regulating emotions. The student frequently argues with the teacher and peers and starts conversations that are not appropriate for class. The teacher works with the student to develop a plan that outlines expected language and behaviors, consequences for inappropriate language and behaviors, and rewards for demonstrating appropriate language and behavior. The plan that is developed is individualized for this student. This strategy is an example of

behavior contract

Jackson is a student who has displayed higher-than-average intelligence and a desire to succeed; however, his classroom performance and participation do not reflect this. Which of the following would be most effective for encouraging Jackson?

breaking tasks into smaller steps and providing immediate feedback

At the beginning of the school year, a middle school teacher gives students note cards and asks students to write one thing they would like to share about themselves and one question they have for the teacher. The teacher responds to one question from the note cards at the end of each class period without sharing any of the personal information of the students. Which of the following describes the primary purpose of this activity?

building a sense of community

A student asks his teacher to help him become better organized. After discussing the situation with the student and offering strategies to improve the student's organization, the teacher offers to meet with him once a week for a few weeks. This is most likely to benefit the student in which way?

by helping him understand and be able to monitor his progress in implementing the new organizational strategies

A teacher has requested a parent conference after a typically high-achieving student has not been completing work. How should the teacher begin the conference?

by sharing positive attributes of the child

An elementary teacher is grouping the class for an upcoming activity. The greatest emphasis should be placed on grouping students in a way that allows for:

challenging and supporting all students.

Ms. Broderick displays a timer in the front of her prekindergarten classroom. Every day, at 9:30am, she announces, "In five minutes it will be time to clean up," and asks a student to help her to set the classroom timer. When the timer rings, she begins singing the clean up song, as the students put everything away. By doing this, Ms. Broderick is demonstrating her understanding that:

children benefit from consistent schedules with clear transitions.

An English Language Arts teacher is planning a unit on literary genres. During a lesson on prose and poetry, the teacher guides students to complete a Venn diagram. This diagram enhances student learning by:

clarifying complex ideas.

The principal is observing Mr. Stein's mathematics class. During the class, Mr. Stein gives the students several problems to complete to practice the concept they've just learned. A handful of students finish the task in under three minutes, most students finish in about five minutes, and some take longer. The students who finish early start chatting with one another at their desks as they wait for their classmates to finish. After ten minutes, when all students have finished, Mr. Stein reviews the answers. In Mr. Stein's evaluation, the principal suggests that Mr. Stein might benefit from training in:

classroom time management.

Which of the following can be communicated through an open forum where all students, parents, and teachers can view the information?

classroom-specific homework assignments and due dates

A view held by _______________ is that children do not learn language through stimulus-response connection, such as how the eye blinks when a puff of air hits it. Instead, this viewpoint would place importance on allowing for processing of language before producing it.

cognitive scientists.

The English and social studies teachers at Calhoun Middle School developed an interdisciplinary instructional unit where students will create a multimedia presentation on literary works by wartime soldiers. The teachers are concerned that many students will focus on the aesthetics of the presentation and not the quality of its content. The most effective strategy to address the concern would be to:

collaborate with the students to design a rubric of expectations upon which the students will be graded that defines the weight of the design in the grading process.

Which of the following is the best way for a teacher to improve the techniques they use when working with English-language learners (ELLs)?

collaborating with an ESL professional

Mr. Herman is a new principal at Hardy Elementary school. During his training period, he learns that Hardy Elementary school has a 25% new teacher attrition rate. Mr. Herman, shocked that this number is so high, vows to provide more support and encouragement to new staff. To do this, he plans to: Create a detailed teacher orientation program to assimilate new teachers to the school and community culture Develop a high-quality teacher mentoring program to support, guide and encourage new teachers Dedicate more resources to collecting teacher feedback at consistent points throughout the year Develop school committees in which teachers can collaborate with one another Seek ways to be more progressive and provide ongoing support If beginning teachers would like to improve their knowledge of how to scaffold lessons for English-language learners, what should Mr. Herman recommend?

collaborating with an ESL professional at the school

Mrs. Hanson wants to create a classroom environment that fosters learning. She decides to rearrange the students' desks and begins to draw up two separate diagrams for how she can organize the desks throughout the classroom. The first diagram has the students' desks organized into four rows of five where each student faces the whiteboard of the classroom. The second diagram has the students' desks organized into five groups of four desks where each group of desks faces each other to form a square and the groups of desks are placed strategically across the classroom. Which of the following activities would be the least suited to Mrs. Hanson organizing the classroom using the first diagram?

collaborative learning projects among groups

Teachers should reflect and address areas of improvement throughout their career. Which of the following is the least helpful in guiding a teacher's focus when clarifying areas of improvement?

comparing the grades of one year to the next

Mrs. Lilly, a fifth-grade teacher, is explaining to her students how the moon influences ocean tides. She uses a visual aid of a ball and two buckets of water to highlight the differences and causes of a high and low tide. Mrs. Lilly uses the visual aid to enhance student learning because her students are probably at what stage of development in terms of Piaget's theory?

concrete operational

Students in a science class make observations prior to beginning a lab. They record 2 to 3 observations and then write a hypothesis to test. Some students struggle with writing a testable hypothesis. Which stage from Piaget's theory of cognitive development best explains where these students are functioning?

concrete operational

Which of the following best describes Piaget's cognitive stage in which children begin to classify ideas and objects and understand conservation?

concrete operational

Mr. Maiden feels that the emphasis that the state places on its annual testing is causing unnecessary stress on his students. He feels that if less weight was placed on one exam, the students would be less anxious and perform better. What would be the best way for Mr. Maiden to advocate for his students?

conducting research and presenting information to the local school board

A second grade teacher does a demonstration where she fills the same cup with water and then pours it into different-shaped containers. One container is tall and thin, while the other is short and wide. She asks students which container holds more water and some students answer correctly while others choose the taller, larger container. What concept is the teacher evaluating?

conservation of mass

The best way for a first-year third-grade teacher to familiarize themselves with the curriculum they will be teaching is to:

consult with other experienced third-grade teachers at the school.

Which of the following strategies best supports a teacher's goal to promote positivity in the classroom?

contacting each parent with a message about their child's strengths

Ms. Morris wants to educate students about the importance of using credible websites in their research. Which of the following characteristics should she instruct students to look for when choosing a website to cite?

contains a published author, date of information, source of the information, and domain address

Johnny has documented learning disabilities and he receives special education and related services in the regular classroom. His teacher, Ms. Spencer, has been following the accommodations and modifications noted in Johnny's IEP, but she is concerned that the accommodations and modifications of Johnny's IEP are not appropriate for him. The annual meeting of the ARD Committee is not scheduled for another three months; however, in consideration of Ms. Spencer's concerns about what she considers inappropriate accommodations and modifications, she should:

continue to use the accommodations and modifications required by the IEP but also request for the ARD Committee to meet before the scheduled annual meeting. Then, the committee will have the opportunity to reevaluate Johnny's IEP.

In summarizing an instructional unit, it is most important for the teacher to:

convey the importance of the information and content being taught, as well as help the students see the content as a coherent whole.

Two children are playing together on the playground. One pretends to be a cat, and the other pretends to be the cat's owner. The two play a game where the owner feeds, pets, and plays with the cat. These children are demonstrating characteristics of which stage of play?

cooperative play

A teacher has chosen an interdisciplinary unit on hot air balloons. Students will study volume in relation to diameter, the role of hot air balloons in history, the impact of temperature on air density, and read non-fiction texts. Which of the following is allowed under copyright law?

copying a short story to be distributed and used in class

Mr. Driscoll is in the process of applying for a teaching job at Smith Middle School. At his last job, Mr. Driscoll listed the class syllabus as well as homework assignments online for students to access at home. Mr. Driscoll received and answered questions about specific assignments online and even posted grades as soon as students completed an assignment. In his job application, what can Mr. Driscoll include as having experience with?

course management systems

What is the highest level of Bloom's taxonomy that a teacher could use?

create

The seventh-grade teachers at Malcolm Middle School have a collaborative meeting each summer to discuss how best to implement the interdisciplinary units for the upcoming school year. The first step the teachers should take in planning the units should be to:

create a clear, overarching outline to identify skills, themes, or concepts that overlap between subject areas.

A second-grade teacher completes a hands-on social studies activity where students work in groups to draw a historical figure from a given time period. The teacher notices her class is much more engaged with the content during this activity but has difficulty defining roles within groups and working collaboratively. In a future lesson, the teacher should:

create roles within groups and have the class create group norms before beginning.

Which of the following is the highest level of thinking according to the modern Bloom's Taxonomy?

create: the ability to produce original thoughts or ideas

Which of the following has been proven to promote a child's sense of safety and security?

creating a predictable daily routine

A new teacher has received an information form for each student that states the family's preferred method of contact and their availability. With this in mind, how should the teacher go about scheduling parent-teacher conferences?

creating a schedule and then offering each parent a day and time that falls into their preferred schedule

A seventh-grade student has difficulties with math computation. What assessment would be most appropriate to determine his needs?

criterion-referenced assessment

What type of assessment compares a student's knowledge or skills against a predetermined standard or learning goal?

criterion-referenced assessment

An 8th-grade teacher is preparing her history class to debate. She tells the students that disagreeing with opponents is necessary in a debate, but it's important to do so respectfully. To ensure that the students disagree with one another respectfully, what should she do next?

demonstrate how she can respectfully disagree with a controversial statement

Mr. Scott is teaching his class about weather and climate. Which of the following activities would be the most likely to encourage his students' critical thinking skills?

designing a travel brochure for a landmark, including climate-appropriate attire and activities

The primary function of a summative assessment is to:

determine student mastery of standards.

While researching for information to use on an upcoming persuasive essay, the writer identifies the author or organizations responsible for the website, then considers possible biases that may influence the information presented. This approach demonstrates an understanding of which part of the research process?

determining source credibility

Use the information below to answer the questions that follow. The information presents a hypothetical classroom scenario, and the questions require that you make the best decision, based on the presented information. Ms. Wright is a 6th-grade math teacher who likes to design her own task-based lessons to go along with the school curriculum. For a unit on statistics, she designs a project in which students create statistical questions, survey others, display and analyze the results and report their interpretations and conclusions in a presentation to the class. After each step, Ms. Wright has the students turn in their work so that she can identify any errors and provide constructive feedback. When the students present their conclusions to the class, Ms. Wright is impressed by her students' presentations and findings but worried because the task has taken two days longer than it should have. Now, she feels pressure to cut down the content in the next unit to get back on schedule. Ms. Wright tells her mentor teacher about her concern, and together they list the specific problems that were causing her delay: 1) Too much time was spent collecting student work and returning it to students. 2) A lot of time was spent commenting on student work and providing feedback. 3) Students often misplaced parts of their assignment, as it was constantly being transferred back and forth from student to teacher. Ms. Wright's decision to leave comments at each stage of the assignment, rather than providing explicit corrections, benefits students by helping them to:

develop a sense of ownership of their work.

When doing group work, a first-grade teacher groups her students randomly, typically resulting in heterogeneous groups. But, today she has chosen to group them homogeneously. What might the teacher be trying to achieve in grouping the students in this way?

differentiation to work on a skill

An upper elementary teacher is giving a test that includes an essay question. One student will receive an accommodation of providing an oral answer to the essay question rather than a written one. Which of the following learning challenges might this student have?

difficulty with written expression

A social studies teacher has students work together to create a "smart" city. As part of the assignment, students must interview someone from City Hall and the teacher helps make arrangements. This approach to learning is considered:

discovery learning.

A social studies teacher wants to create a living museum in which students dress up as historical characters and give mini-presentations when visitors approach their station. The teacher would like to invite the community and parents. Which of the following should be the first task to complete on an action plan for implementing this project?

discussing plans with campus administrators

Mrs. Clark wants to divide her class into eight sections with each section having an activity center where her students can do a hands-on science experiment to correlate with the current science instructional unit. When designing the activity centers, it is most important for Mrs. Clark to ensure that:

each activity clearly relates to and supports a specific concept from the instructional unit.

Which of the following would not negatively impact a student's physical development?

emotional abuse

Sara is a student who reads above grade-level and discusses ideas at a high level, but has been known to neglect class work and leave assignments unfinished when she does not see the point. She is not motivated by grades and her participation depends mostly on the subject being studied. Which of the following best describes Sara?

intrinsically motivated by interest in the content

Mrs. Santiago's eighth-grade class is planning a cultural fair to showcase the variety of cultural backgrounds at their middle school. As a celebration of their traditions, parents and students will prepare food, play music, dance, and wear traditional attire from their own cultures. Which of the following best describes the reason that Mrs. Santiago considers it important for her class to plan this event?

encouraging diversity awareness among students

The Pierce School District has noticed that the parents of the middle-school ELLs aren't often involved in school activities. The school administration has decided to put more effort into building trust amongst parents and making them feel welcome. They should try all of the following except:

encouraging parents to interact with their children as much as possible in English.

Which of the following is not a strategy for externally motivating students?

encouraging students to enjoy learning about new concepts

Which of the following best describes the goal of using portfolios as part of assessment?

encouraging students to evaluate and reflect on their own work

Which of the following activities could be implemented in an elementary school classroom to appeal to students' need for peer approval?

encouraging students to use kind words and praise their peers' efforts when trying new or difficult tasks

A first-grade teacher sends home an outline of an upcoming unit. Grade 1st Unit Title Surviving in the Wild Standard Addressed 1.9 Organisms and environments. The student knows that the living environment is composed of relationships between organisms and the life cycles that occur. 1.9(C) Gather evidence of interdependence among living organisms such as energy transfer through food chains or animals using plants for shelter. Day Learning Goal Activity Assessment 1 The learner will categorize organisms. Classify organisms according to living/nonliving. Exit ticket matching activity 2 The learner will examine an organism's environment. Read and take notes on various habitats. Fill-in-the-blank worksheet on habitats. 3 The learner will determine how an organism survives. Make food chains in a small group using labeled pictures. Draw a food chain as a group based on a habitat assigned by the teacher. 4 The learner will create a food chain. Work independently to create a food chain. Students create their own food chain. By sending home this unit plan, the teacher is encouraging the parents to:

engage in conversations with their children about what they are learning in class.

A first-grade class is working outside in the school garden, and the teacher notices most of the students want to work on the same five plants. Many students are starting to argue and take garden tools away from each other. Which of the following approaches should the teacher take to increase students' interpersonal skills?

engage students in a class discussion about possible solutions for their disagreements

Technology is best utilized in the classroom when it can accomplish which of the following?

enhances the learning objective

A diverse elementary school has set a goal to increase parental involvement over the course of the school year. The leadership of the school would especially like to engage parents of English learners (ELs). To help meet this goal, a third-grade teacher has taken several steps to invite parents to become part of the classroom learning environment. The teacher has sent home monthly newsletters, kept the class website updated, and called parents regularly. Despite these efforts, only 10 parents came to a recent open house. To establish better parent-teacher relationships, the teacher should:

ensure parents are able to receive, read, and understand all forms of communication coming from the school.

When planning instruction, which of the following should not be a priority in the pacing and planning of daily lessons?

equally distribute time across all objectives

A positive learning environment requires the teachers to have high academic expectations, students that are developmentally responsive, and which of the following to perform successfully?

equitable resources

Which of the following is the best way to prepare for potentially challenging situations with students throughout the school year?

establish positive relationships by getting to know students and their families at the start of the school year

Which of the following assessments is formative?

exit slips

Which of the following involves students writing a brief response to a prompt about the day's topic before leaving the classroom?

exit ticket

A new teacher wants to observe experienced teachers and their instructional methods. Which of the following would be the greatest benefit of this practice?

expanding their knowledge of classroom management skills and how to implement them

A teacher has an eighth-grade science class with a wide range of student abilities. With this in mind, which of the following is the best way to present physics concepts?

explain abstract concepts using mathematical equations and include a physical demonstration

Developmentally appropriate traits of five year old children include the ability to

express feelings, follow three-step commands, and copy basic shapes

A seventh-grade student completes all of the homework assignments during a unit and studies for the test. She makes an A on the test. Reflecting on her grade earned, she decides to continue this work pattern for the next unit. What type of reward system is at play?

extrinsic motivation

Under the Individuals with Disabilities Education Act (IDEA), public and charter schools have an obligation to:

find, test, and provide necessary disability services to children from birth to transition.

Which of the following would be a developmentally-appropriate activity for typical students?

first-grade students using manipulatives to understand addition and subtraction

A school administrator observes a third-grade classroom and notices the same five students answer a majority of the questions posed by the teacher. When the administrator visits with the teacher, the teacher comments that those five students are part of the gifted and talented program. Which of the following steps should the teacher take to create a more equitable classroom environment?

institute additional wait time after questions to give all students an opportunity to formulate a response

Mr. Valdez planned a revision lesson where he shows the class before and after examples of the rough and final draft of an essay. He then provides class time for students to begin revising their work, but many students are emphatic that their essay is done and want to turn it in as is. Mr. Valdez backs up and models the process of revision, and writes a list on the board, specifying what to look for as they revise. What does Mr. Valdez demonstrate in this scenario?

flexibility and the ability to adjust a lesson that is not working as intended

A middle school teacher has found a published lesson plan with activities and quizzes online and plans to use these materials for an upcoming lesson on weather patterns. When planning to use published materials for instruction, it is important for the teacher to:

follow copyright requirements for the use of published materials.

Standardized tests, IQ tests, end-of-unit exams, norm-referenced tests, and achievement tests are all examples of:

formal assessments.

A teacher writes a different question at the top of several pieces of chart paper. Students move in groups to each chart and write their answers or elaborate on other groups' answers. The teacher observes student responses to guide the discussion questions when the activity concludes. What type of assessment is this?

formative

After giving a lesson on simple sentences, Mr. Hummel posts a single review question and he asks the students to answer this question on an exit slip before heading off to lunch. What form of assessment is Mr. Hummel using?

formative

Students are working to solve the following question: ½ - x = ¼. The teacher then gives the following as an example: "If you are sharing a pizza with somebody and there is half a pizza left, how much must the other person eat so that you only have one quarter of the pizza left?" As the teacher engages with several students, the teacher observes students are still having difficulty understanding the concept of fractions. The teacher then uses a pie chart to help explain the concept. Which of the following types of assessments has the teacher used?

formative

In the middle of a unit on lunar cycles, Mr. Rice hands out drawing paper and gives his students the following task: A new moon is always high in the sky at noon. Use what you know about the positions of the sun, moon, and Earth during a new moon to draw a diagram to show why a new moon is in the sky at noon. After the students finished their drawings, Mr. Rice looked at them before he planned the next unit in the lesson. Mr. Rice used this activity as which of the following?

formative assessment

Which of the following would be the best tip for having positive communication with parents and families?

frequent communication is key

Which of the following is NOT a characteristic of an appropriate professional development goal?

general

Mr. Dwight, a high school teacher, nominated his student, Frank, for an award that recognizes outstanding achievement. Frank has Autism Spectrum Disorder and overcame many obstacles to make the Honor Roll for the first time. Frank won the award, and the local newspaper asked Mr. Dwight to submit a write-up about how Frank has overcome the obstacles associated with his disorder to achieve such an honor. What must Mr. Dwight do before submitting the article?

get Frank's parent and/or guardian's permission for the article to be published

Which of the following exceptionalities does Section 504 of the Rehabilitation Act of 1974 not protect?

giftedness

Ms. Blake is a special educator who works with a student named Mark. Mark has been doing well on the multiple-choice and fill-in-the-blank sections of his science exams, but struggles with the essay section. Ms. Blake observed a science class and can confirm that Mark clearly knows the material based on his comments. To best advocate for Mark, Ms. Blake should ask his science teacher to:

give Mark the essay section orally.

A teacher overhears a conversation between two children during center time. J'Mari: "I don't want you to play with me!" Emily: "But I want to play with the blocks too!" In this situation, the teacher should:

give options to the two students, such as playing in different areas of the center or choosing a collaborative activity together.

Which of the following is an effective way for the bilingual community to get involved with ELLs?

giving presentations about their experiences using more than one language to help them in their professional lives

As a social studies teacher, Mr. Ray strives to return daily work the next day with markings to indicate areas in which students can improve their performance. The greatest benefit to Mr. Ray's practice is:

giving students immediate feedback to increase their motivation and ability to continually improve their performance.

Which of the following best promotes professional collaboration and mutual support?

grade-level teachers meeting with elective teachers to integrate curriculum

A teacher is planning the next unit and wants to target specific areas in which students need additional instruction. Which of the following could help identify areas of need?

gradebook

Mr. Jolly wants to create a positive climate for his classroom. Which of the following actions would best promote a positive classroom climate?

greeting each student as they enter the classroom

The primary function of a formative assessment is to:

guide ongoing instruction.

A third-grade teacher receives a new program for project-based learning on her classroom devices that allows students to illustrate and/or add text to a graphic. An example of a developmentally appropriate activity that best demonstrates the application of this technology to all individuals in her classroom would be to:

have groups of students complete a representation of the chapter book they are reading in their preferred format (illustration, graphic image, report, etc.).

As the second-grade teachers are planning their next unit, they focus on differentiation for individual students. What is the best method for reaching students?

having centers with different level activities and each child works on their level

Research shows that de-emphasizing grades can increase student motivation, participation, and confidence. Which of the following best demonstrates this practice?

having students answer questions on mini wipe boards to check for understanding

Students in third grade are studying 2-D and 3-D shapes. The teacher wants to use technology to support students' understanding of geometry. Which of the following is the most appropriate and effective use of technology for this purpose?

having students explore a website that allows users to construct and deconstruct 3-D figures using 2-D figures

According to Maslow's theory of the hierarchy of needs, a person cannot achieve a sense of belonging without first:

having their safety needs met.

An elementary teacher observes that some students use their fingers to count, add, and subtract after showing mastery of these skills on a test. The teacher should be aware that this act:

helps students guide their own thinking.

When a teacher asks students to analyze the author's point of view and motivation for writing a piece, they are developing:

higher order thinking skills.

Ms. Henricks finds that much of class time is spent responding to student requests for bathroom breaks and asking students to quiet down. These interruptions disrupt the classroom routine and take time away from instruction. What could Ms. Henricks do to ensure her instructional time is not lost to these interruptions?

instruct students to use and follow nonverbal cues

Who is generally responsible for creating the scope and sequence of a curriculum?

instructional facilitators at the district level

Which is the most important item to keep in mind when choosing an internet article to reference in a research paper?

how credible, reliable, or accurate the source is

Students are conducting research before drafting a research paper. The teacher demonstrates how to use the online search features to find a source. Which step should the teacher model next to ensure students are locating credible sources?

how to review a chosen source's bibliography information

While giving a lecture, a teacher has stopping points where she asks multiple-choice questions and all students respond through the use of a clicker. This immediate feedback allows the teacher to:

identify misconceptions and address them.

Which of the following is the primary reason that teachers administer formative assessments?

identifying mastered skills and skills in need of further development

A teacher is having individual conferences with students to discuss progress on their Lewis and Clark project. During each conference, the teacher will identify errors and provide specific feedback as to how they can improve. Which of the following should the teacher also incorporate into the conferences?

identifying positive aspects of the student's work

Specific, teacher-assigned roles would be most effective for elementary students working:

in lab groups.

In order to promote independent learning, a teacher must begin instruction:

in the zone of proximal development.

A middle school teacher planned a lesson that went quicker than anticipated. After the lesson finished, students were eager to continue the discussion and learn more. How could the teacher modify this lesson with the next class?

incorporate more discussion questions into the activity

Which of the following would be appropriate for a general education teacher to do to assist an intermediate-level ELL who has been struggling to understand the content?

incorporate more visuals, charts, and graphic organizers into the lessons

A teacher sends home a monthly communication that reviews projects and topics covered during the past month, upcoming units of study, and supplemental resources that can be used for remediation and enrichment. At the end of the communication is a survey that allows parents to request a conference, sign up to attend a "homework help" night for parents/guardians, volunteer at school events, or donate supplies for upcoming projects. Which of the following describes the greatest benefit of this survey?

increased parent involvement

When teaching social studies, Ms. Linder is hoping to encourage self-directed learning. What result is she hoping to achieve with this focus?

increased student ownership of learning

It is important for teachers to make time for group work in the classroom because group work, when done effectively, does all of the following except:

increases student feelings of autonomy

A class is learning about the different eras of innovation, from the Industrial Revolution to modern times. As part of the unit, the teacher asks students to choose any invention that has influenced their lives in a drastic way and write about the history of that invention. She gives the students a choice because giving students a choice:

increases their intrinsic motivation

A mathematics teacher gives her class a two-question clicker quiz at the end of each class period and tabulates their answers according to their mathematical understanding, misconceptions, and error patterns. If her goal is improvement in her students' mathematical proficiency, her best use of the data would be to use it to:

inform upcoming instructional strategies.

A teacher is monitoring her class while the students are involved in a group activity exploring the size of angles in a set of triangles. She moves from group to group, pausing and watching the group dynamics. What the teacher is doing can best be described as:

informal formative assessment.

Which of the following activities requires students to use reasoning skills and higher-level thinking?

inquiry-based activities

Mrs. Sanders gives her high school English students the opportunity to choose the topics of their literature papers. Which of the following types of motivation does Mrs. Sanders hope to give her students by allowing them to choose the topics of the literature papers?

intrinsic

Providing students choices in their learning promotes which of the following?

intrinsic motivation

A second-grade class is reading an article while studying magnets. The teacher projects the article while each student reads along on a printed copy. Before reading, the teacher performs the following think-aloud. So, we're about to read the article "Sticking to Magnets" by Timothy H. Allenthorpe, Ph.D. Ph.D. is an acronym for Doctor of Philosophy. That means that this author went to school for a very long time and studied extensively in his field. This author must know a lot about magnets. If I look over here at the "Meet the Author" section of our textbook, it says he graduated from Columbia University. That's pretty impressive. It also says that he helped NASA develop a type of magnet used on space shuttles!" The primary purpose of this think-aloud is to:

introduce students to the process of determining a text's credibility by analyzing the author's expertise and credentials.

The teacher of a high school ESL class notices that several of the students in the class are lacking confidence in themselves. What is an effective way that the teacher can help the students to gain a sense of empowerment connected with being bilingual?

inviting college students to speak to the class about their experiences studying abroad and the benefits of knowing two languages and cultures in the current job market

A high school English teacher has been thinking of ways to support his English-language-learning students. To do this, he wants to involve families in the students' learning process. Which of the following activities would best facilitate this?

inviting parents to a school event in which they can learn different strategies to use at home to support their student

Which of the following activities best promotes family-centered programming with the school?

inviting students to share a family tradition or custom and allowing all students to participate in it during a school day festival

Mr. Ray's seventh-grade class continually disrupts instruction and is proving difficult to manage. Mr. Ray decides to create a system where each disciplinary infraction results in the class earning a checkmark. If, at the end of the week, the class has below a certain number of checkmarks, they receive a class-wide reward. This classroom management strategy is likely to be a success because:

it creates an incentive for students to behave to meet the approval of their peers.

During a heated debate in Mr. Martinez's government class, a student declares that the problem with government spending is that the Congress spends so much money on themselves that they become unaware of the value of a dollar. Mr. Martinez then addresses the class in a calm voice saying, "It has been suggested that the incentives of Congress voting themselves pay raises and benefits results in a problem of fiscal irresponsibility of the government. Who agrees or disagrees with this statement?" Mr. Martinez's rephrasing of the student's declaration is most helpful because:

it focuses the class on a single topic of debate while providing a framework for response.

A first-year ninth-grade teacher, Ms. Simpson, is planning learning goals for an upcoming instructional unit. One of the goals is that students will be able to enjoy music from various cultures. The most significant problem with this learning goal is that:

it is difficult to objectively assess a student's progress or achievement.

One advantage of using portfolio assessment over traditional quizzes and tests is that:

it provides a reliable composite of progress over time.

The main purpose of educational journals and publications is to:

keep educators aware of current trends and research relating to instructional techniques and best practices.

A teacher is setting professional development goals for the school year using data from the previous year. ELL Gifted/Talented Low SES Fall Spring Fall Spring Fall Spring % correct 44 42 81 84 62 66 Based on the data, which professional development activity would be the most relevant?

language strategies

Which of the following terms best describes a cognitive process that uses indirect and creative approaches to solve problems?

lateral thinking

Which of the following describes the best strategy to establish behavioral expectations at the beginning of the year for a class?

leading students in a discussion in which they develop a mutual list of class behavior expectations with examples for each item on the list

A teacher uses backward design when planning to ensure that she teaches all necessary components of a concept. Which of the following should the teacher create first when using this approach to lesson planning?

learning objectives

A science teacher wants to try a new educational social app for students to engage and help each other with concepts. They can post questions to a private board and respond to others. The teacher will monitor exchanges. Which of these people on campus would be most helpful in planning the use of this app?

library media specialist

A language arts teacher writes the following learning objective for the day's lesson: The students will learn about the differences between similes and metaphors. How should this teacher revise the learning objective?

make the desired outcome measurable

The best way that schools can provide one-on-one assistance for new teachers is through:

matching new teachers with mentors.

A first-grade teacher spends much of the first week practicing routines and procedures. Part of the practice is learning specific behaviors for transitioning between activities. Which of the following is the primary goal for this routine?

maximizing time on-task for the class throughout the year

Which of the following describes the best use of a norm-referenced test?

measuring performance in a particular area in comparison to same-aged peers

What is the primary goal of summative assessments?

measuring student achievement

Which of these is not considered a risky behavior for a high school student?

missing Saturday morning practice due to oversleeping

A teacher wants to incorporate more interactive online tools into his curriculum. During the spring break, he provides his 8th-grade students with a research task to complete online. The task involves surveying classmates, so he includes the link to an interactive discussion forum, and encourages the students to use it to interact with one another. However, most of the students opt for more traditional methods of communicating with classmates, such as in person or through text message. What should the teacher have done to encourage the students to use the new technology?

modeled how students could use the forums in an optimal way

A checklist would likely be most useful and appropriate for assessing which of the following goals in elementary?

monitoring students' use of science safety skills during a lab

Mr. Jones is creating an assessment for his seventh-grade English class, and he wants to utilize all levels of Bloom's taxonomy within the assessment. Which of the following assessments should he choose?

multiple choice test with varying levels of rigor in the questions

A science teacher is planning an instructional unit on the laws of physics. The teacher identifies five afterschool activities that students most commonly participate in and designs learning experiences that correlate to each after school activity. The teacher allows each student to choose from the five learning experiences based on their interest in the activities. This instructional design best reflects the following learning principle:

new ideas and concepts are better reinforced when students process information in the context of previously known information and experiences.

When creating learning centers for a school's early education students, which of the following locations would be most appropriate for an animal-learning center?

next to the science lab

Ms. Smith follows a particular theory of child management. In her classroom, you will see: Ms. Smith does not discipline students often. She does not want to punish or impede a student's natural desire to improve. Students manage their own behaviors with subtle guidance from Ms. Smith, such as eye contact or a gentle pat on the shoulder when misbehaving. What theory of child management does Ms. Smith employ?

non-interventionist

During a lesson on variables with exponents, Ms. Wiley notices two students whispering and passing notes back and forth. Without interrupting the lesson, she moves and stands near these students. This action can be classified as:

non-verbal communication.

A teacher is concerned about a student who seems to be behind peers in reading. Which of the following types of tests will provide the most accurate and relevant information about the student in comparison to peers?

norm-referenced

A fifth-grade teacher is concerned about the lack of academic achievement for one of her students. The teacher requests that the special education teacher observe and conference with her regarding appropriate instructional strategies. What is the best response that the special education teacher can offer?

observe and recommend the RtI process

Which of the following would be the best way for a new teacher to improve his classroom time management skills?

observing effective techniques used by experienced teachers and building upon their expertise

Bobby was a hardworking, respectful tenth grader until a few weeks ago when his teacher began noticing his grades were slipping, he was having trouble concentrating during class, and he was easily irritated by other students. Out of concern, the teacher asks Bobby to stay after class one day to discuss his slipping performance. Bobby confides in the teacher that he has to work two part-time jobs because his father was recently laid-off and he knows the stress is hurting his academic performance. Later in the week, Bobby's mother calls and reveals to the teacher she is at a loss of how to alleviate the pressure Bobby feels. The teacher can best respond to Bobby's mother by:

offering to meet with her to discuss how the two can work together to balance Bobby's school work and his after-school obligations.

Which of the following is the most effective way to report student progress?

online gradebook with individual logins for each student

Mr. House has a behavior board in his second-grade class. At the end of the day, students get a 1-5 (5 being the best) score based on their behavior for the day. If the student has 23 points or higher at the end of the week, they will earn a reward; if they have less than 23 points, they will not. Which of the following theories does this technique reflect?

operant conditioning

In science class, a student rarely remembers to complete homework and earns a low average grade on quizzes and tests. During discussion, the student asks thoughtful questions and participates eagerly. Which of the following assessment formats would allow the science teacher to fully evaluate the student's mastery of content?

oral questioning

Mr. MacArthur is about to assign a major project to his students. Which of the following strategies would help promote students' understanding of the project's expectations?

providing and discussing the rubric that will be used to grade the project with the students

In the past, Mr. Garcia felt like he hadn't been able to establish a strong relationship with his students. This year, his goal is to create a positive classroom environment in which he has strong, trusting relationships with each student. Which of the following activities could Mr. Garcia try on the first day of school to best support this goal?

participating with the class in an ice-breaker game, in which personal interests are revealed

In order to increase students' motivation to learn math concepts, a teacher could implement:

peer tutoring in which every student has the opportunity to tutor and be tutored.

A teacher needs to assess mastery at the end of a social studies unit. Rather than giving a multiple choice test, she has students create a newspaper that covers the major events from the unit. What type of assessment is the teacher using?

performance assessment

Which of the following assessments would be best for measuring a student's ability to apply concepts learned in a unit to the outside world?

performance-based

In order to evaluate the effectiveness of instruction when introducing a new unit, a teacher should do which of the following?

periodically question the students during the presentation

Family Education Rights Privacy Act (FERPA) gives parents access to all of the following except:

personal notes from teachers.

Maslow's theory detailed a hierarchy of needs. In order for people to meet their full potential they must meet a series of needs in a specific order. If one or some of the needs in one level are not met, it is difficult to begin meeting needs in the next level. Which of the following lists the needs in the correct order (high to low)?

physiological needs, safety, love/belonging, esteem, self-actualization

A kindergarten teacher sends a parent survey home to learn more about the interests and cultural background of her students' families. She finds that her students come from many different cultural backgrounds that she is unfamiliar with. One way the teacher can engage these families would be to:

plan family show and tell days, where students and parents can give presentations about their family and culture to the class.

A kindergarten teacher notices students are very animated and have difficulty settling down after lunch and recess. Which of the following steps could the teacher take to help students refocus for instructional time?

play calming music for students while they lay their heads down on their desks

At the end of a unit, the teacher has students turn in all of their lab write-ups and work related to the unit. What type of assessment does this represent?

portfolio

Students maintain a writing journal throughout the semester. They keep graphic organizers, rough drafts, and final drafts in it. At the end of the grading period, the teacher uses the journal to evaluate growth. What type of assessment is the teacher using?

portfolio

A third-grade teacher asks students to document their learning throughout the school year by choosing exemplary samples of their work. At the end of the year, they are compiled, along with teacher feedback, self-evaluations, and other assessment information. This is an example of a(an):

portfolio.

A small group of students has been scoring below average on math benchmarks. The teacher pulls this group for small group remediation. Which of the following assessments would be most appropriate for this group of students?

progress monitoring

Which is the most likely way to engage a class of elementary students in a lesson on the wave properties of frequency, wavelength, and amplitude while still presenting the best learning opportunity? After giving a few examples:

project pictures of transverse waves of different wavelength and amplitude, while for each, students wiggle fast or slow to indicate frequency, and with large wiggles or small wiggles to indicate amplitude.

An English class has recently finished reading a chapter of Ender's Game. Mrs. Weyenberg assigns the class an essay in which the students must respond to the story by analyzing the conflict between the main character, Ender, and a fellow character. She asks the students to explain the conflict, write about each character's emotions towards the other character, and tell how the situation could have been resolved without the use of violence. The teacher's assignment of this prompt best:

promotes the students' use of higher-order thinking skills in their writing.

A high school elective teacher has an inclusion class in which the students have significant variations in abilities and needs, including two students who attend class with a 1:1 instructional assistant. When assigning small group activities, the teacher regularly arranges mixed groups which include students with and without disabilities. Which of the following describes the most important benefit of grouping students in this manner?

promoting positive relationships between all students by allowing them to collaborate and connect

Mr. Jacobs has a class of 25 students. Typically it takes him about 5 minutes to complete attendance at the start of each class. With this time lost, Mr. Jacobs is often unable to complete the full lessons he has planned for the class. What should Mr. Jacobs do to avoid this wasted time?

provide students with a task to complete while he takes attendance

Mrs. Parker, an elementary teacher, designs an instructional activity where she will write a common word on the board and a student will think of a synonym of the word. The student will then lead the class in a game of "hangman" with the chosen synonym. In designing this instructional activity, the teacher most likely wanted to:

provide students with an opportunity to be actively engaged in the learning process.

At the end of each unit, a teacher incorporates a day where no new objectives are taught. On that day, students take time to respond to a series of questions asking them to reflect on different aspects of their learning throughout the unit. What is the purpose of this instructional practice?

provide students with an opportunity to self-assess

A first-grade teacher writes out the daily class schedule on the board and places large pictures of the activity next to it. By displaying the schedule, the teacher is:

providing all students with a means to independently prepare for transitions and activities.

A history teacher is planning a unit on The Great Depression. The teacher is developing the following activities for this unit: an analysis of common illnesses and how they impacted the population in the absence of certain vaccines, a review of typical wages and a budgeting activity for a fictional family of six during this time period, and a reflection of themes exhibited in a poem written during the time period. Which of the following strategies is the teacher using based on these activities?

providing interdisciplinary instruction

Which of the following tasks would be appropriate to ask of a paraeducator in a classroom?

providing one-on-one tutoring during independent practice time

A teacher overhears several students talking about an algebra class that they are taking. Here are some of the comments made by the students: "I don't know why I need to use this class, I never use math!" "I'm not planning to go to college, so why do I need algebra?" "This class should be an elective - it has nothing to do with my tech program." Which of the following would be the best response to these comments?

providing real-world examples and scenarios when introducing new concepts in class

An upper elementary teacher is introducing new content to the class. The lesson is given as direct instruction and follows this sequence: activate prior knowledge introduce new concepts guided practice with feedback assign independent practice During guided practice, students are making consistent errors. The teacher can best respond by:

providing simpler questions that students can correctly answer and then gradually increasing the difficulty level

Mr. Kirby is working with his team to develop a lesson for his eighth-grade language arts class. One colleague recommends using scaffolding to help students better access the information. Which of the following strategies best uses this approach?

providing students with a graphic organizer to fill in as they read material and listen to lectures

All of the following are ways that schools can reasonably eliminate barriers to family involvement in school affairs except:

providing weekly check-ins on student progress.

During independent work time, a teacher has book chats at the conference table in the room. In order to be available to all students without interrupting the book chat, the teacher should:

put out a sheet where students can write questions that will be addressed between chats.

A student is struggling with behaviors despite multiple interventions. A special education teacher observes the student in various classroom settings over the course of a week. What type of assessment is being conducted?

qualitative

Before reading a science text written in English, a teacher asks her English-language learners (ELLs) to predict what the article is about by reading the title and subtitles and looking at the images and charts. Which of the following skills does this activity develop?

reading comprehension

A second grade teacher is planning a language arts lesson to introduce elements of fairy tales. An appropriate lesson to begin the unit would include which of the following?

reading familiar fairy tales aloud and pointing out key features

Which of the following should be discussed with students on the first day of starting a new unit?

real-world applications of the unit's concepts

There are a handful of students in Mr. Underwood's class who show signs of being at a higher developmental stage than that of their classmates. Mr. Underwood wants to ensure that these students are being challenged. What should he do?

refer the students to a gifted and talented program

A third-grade student with learning disabilities requires constant redirection when working in collaborative groups. The teacher requests the special education teacher begin the initial evaluation process. What is the best response to provide the general education teacher?

refer the teacher to the RtI process

During his science class, Mr. Patterson plays a short video clip explaining the process of forming a raindrop. At the end of the clip, he asks his students to verbally explain to a partner what they heard and understood. As students do this, Mr. Patterson circulates the room to check for understanding and clarify any points of confusion. The main purpose of this activity is to help students:

reinforce the information obtained from the video.

Which of the following strategies provides instruction that supports all levels of cognitive development?

reinforcing challenging concepts with real-world scenarios and opportunities to apply new skills

As part of her professional development plan, A teacher creates a survey for her students about learning styles. This survey is likely to be:

relevant because it focuses on student needs.

Which of the following terms means that a test produces the same scores when given in the same conditions?

reliability

When planning an upcoming unit, the teacher considered the various objectives and the amount of class time that was necessary to teach the content in the previous school year. Which of the following is not necessary to incorporate into plans for the progression of learning objectives throughout the unit?

remediation for students who have fallen behind

As part of a final portfolio, Ms. Johnson asks her students to submit a personal narrative of a memorable childhood experience. Lisa, one of the more quiet students in the class, submits a detailed story about a time when she disobeyed her father and was abused as a result. The story described a time in the past, and Ms. Johnson isn't sure whether Lisa is currently in danger. What should she do?

report the incident to the Texas Department of Family Protective Services

What is the best procedure for providing personalized feedback after giving a quiz?

return graded quizzes with an area of weakness identified

A student in Mr. Andrews' inclusion class has recently been found eligible for special education services. After the ARD meeting, Mrs. Andrews receives a copy of the student's IEP which includes accommodations for extended time and small group setting during tests. Which of the following is the most appropriate way for Mr. Andrews to implement the IEP?

review the IEP and provide the student with both extended time and small group setting for assessments

During a math unit, a teacher plans a weekly quiz as a formative assessment. One week, most students miss the same two questions. The teacher can best respond to this by using which of the following approaches?

reviewing the concept covered on these questions and adding a relevant question to the next quiz

Each day, as second-grade students enter their classroom, they place their homework folder in an assigned cubby. As students begin morning work, the teacher notes absences by checking the cubbies. Which of the following statements explains the reason behind this practice?

routines prevent wasted time

Which best describes a showcase portfolio?

samples of a student's best work are chosen and compiled with teacher input

A middle school has three feeder elementary schools. To ensure that all students feel connected on their first day of school, the science teacher plans small group activities in her classroom. What is the best method to seat students?

seat students in groups of three that include a student from each school

Read the description of Lakesfield School and then answer the questions that follow. Mr. Logan is the principal at Lakesfield School. Lakesfield is the oldest school in Middlebrook County. The community is located about 25 miles north of a big city in a small semi-rural town, centered around a meat-packing plant. Almost a quarter of the students come from migrant families who work at the plant or in nearby farms. More than half of the students enrolled are eligible for free or reduced-price lunches. Approximately 40% of the students qualify as English-language learners. Many parents do not read or write well, but they have high hopes for their children and want them to succeed in school. Mr. Logan understands the importance of involving parents in their children's education, and plans to prioritize this in the upcoming school year. To best facilitate communication with parents of the English-language learners in the school, Mr. Logan should require teachers to:

send messages home in the parents' native language.

Read the description of Lakesfield School and then answer the questions that follow. Mr. Logan is the principal at Lakesfield School. Lakesfield is the oldest school in Middlebrook County. The community is located about 25 miles north of a big city in a small semi-rural town, centered around a meat-packing plant. Almost a quarter of the students come from migrant families who work at the plant or in nearby farms. More than half of the students enrolled are eligible for free or reduced-price lunches. Approximately 40% of the students qualify as English-language learners. Many parents do not read or write well, but they have high hopes for their children and want them to succeed in school. Mr. Logan understands the importance of involving parents in their children's education, and plans to prioritize this in the upcoming school year. Which of the following actions can Mr. Logan recommend that teachers do to promote involvement from all parents in their child's education?

send out progress reports and ask that parents or guardians sign them to monitor their child's learning

School districts should be expected to do all of the following to encourage the parents and guardians of English learners to become more actively involved in their children's education EXCEPT:

send written updates to parents regarding student progress at the end of each week.

Which of these is the least reliable means of communication between teachers and parents?

sending home information with students

A student has been acting silly and disrupting class lately. She needs frequent reminders to be on task and it is impacting her academic progress. What is the most appropriate first action in response to these behaviors?

setting up a student-teacher conference

Read the information below and answer the questions that follow: Ms. Johnson will be starting her first year as a 6th-grade teacher at Plainfield Middle School this fall. In preparation for the upcoming year, she reads through the school's mission statement and finds the following: "Plainfield Middle School believes it is important for children to be involved and enthusiastic participants in their education. Thus, we encourage students to exercise curiosity, critical thinking, and creativity in an active, collaborative learning environment. Our teachers demonstrate expertise in their fields and a passion for lifelong learning." In which of the following ways could Ms. Johnson demonstrate her passion for lifelong learning?

sharing recommendations of books she has recently read on the classroom bulletin board, alongside student recommendations

Which of the following question formats would be most appropriate for assessing student mastery of the following objective? Students will explain how gravity and energy influence the formation of a galaxy.

short answer

A teacher wants students to access a password-protected website at home for an assignment. Before giving out login credentials, the teacher should consult the:

site license guidelines.

Which of the following is a learning technique that incorporates increasing intervals of time between practice of previously learned materials?

spaced repetition

Mr. Buffett is a high school home economics teacher. He wants his students to become familiar with household budgets. Which would be the most appropriate software method to use for this assignment?

spreadsheets

Which of the following is a factor in students from diverse cultural backgrounds' education that is not influenced by a teacher's own cultural values, beliefs, and attitudes?

standards and curriculum

A second-grade teacher is computing grades. Rather than working for typical letter grades, students are working for mastery of each concept. Each subject is broken into small tasks identified as learning goals. Students are given a grade for each task on a scale of 1 to 4, with 1 representing little understanding of the concept and 4 representing an understanding greater than expected. What type of grading is the teacher implementing?

standards-based grading

An elementary teacher just received her roster for the new school year. She would like to know her new students' current levels of performance in math. Which of the following should she consult?

statewide achievement test item data

A teacher has identified the state standards associated with a unit of study. Of the following, what is the most important consideration to make as she now plans activities for the unit?

student needs, interests, and abilities

An ESL teacher frequently checks on her students listening and speaking skills. After her history lesson, she pairs students to discuss what was taught and takes the time to do an informal check. While walking around she overhears an ELL student speak with some language errors to his group. Which practice would benefit the student while making sure the student keeps taking risks in speaking?

student-led conference

Which of the following is the best approach to target individual skills that a student needs to develop?

student-teacher conference

Ms. Hanson, an eighth-grade science teacher, notices one of her students has begun to talk frequently to her friends while conducting an experiment and is not fulfilling her responsibilities. In addressing the situation with the student, Ms. Hanson should keep in mind that:

students at this age are concerned about their peers' perceptions of themselves, and they have an overwhelming desire to conform to their classmates.

Mrs. Langston is an English teacher and requires her students to keep a portfolio of their writing assignments, which includes drafts and final copies. The greatest benefit to the portfolios is that:

students can view how their writing has improved over time.

Which of the following would be considered a summative assessment in a social studies classroom?

students creating a travel brochure for a city after studying the culture of Germany

Mr. Lopez is introducing an instructional unit on World War II to his students. Before he begins an instructional activity, he asks each student to write five questions they have about World War II. At the end of the instructional unit, he returns the five questions and has the students provide answers to their questions. A primary reason for Mr. Lopez utilizing this instructional strategy is to help:

students gain ownership of the learning material, and maintain student interest in the instructional concepts.

As a first-year teacher, Mr. Kramer is being observed in his classroom by his principal. One of the observations the principal makes is that Mr. Kramer demonstrates higher expectations to higher achieving students and has lower expectations for lower achieving students. This observation has the greatest effect on the students in that:

students respond to expectations, and low expectations will reinforce to low achieving students that they are less able to achieve.

Which of the following is a diagnostic formative assessment?

students take a pre-test on Ancient Greece

Mr. Jameson believes that student self-evaluations are important as they encourage students to monitor their own learning needs and serve as an additional source of information on student learning. As an additional way to monitor ELL students, he uses "exit slips" in which:

students write a response to a teacher prompt or question on a card or slip of paper at the end of class.

The semester exam administered to students at the end of the term is considered to be a:

summative assessment.

A teacher sends home a weekly newsletter on Friday about the upcoming week. Each student writes on the newsletter one thing they learned and one thing they did well. The teacher also writes a note about accomplishments and concerns. The newsletter is returned signed on Monday. The primary reason to send this newsletter is:

supporting communication between home and school.

A kindergarten teacher wants to demonstrate the concept of centrifugal force. The best instructional approach would be to:

swing a bucket of water overhead in front of the students and then explain why the water did not fall out.

Read the information below and answer the questions that follow: Ms. Johnson will be starting her first year as a 6th-grade teacher at Plainfield Middle School this fall. In preparation for the upcoming year, she reads through the school's mission statement and finds the following: "Plainfield Middle School believes it is important for children to be involved and enthusiastic participants in their education. Thus, we encourage students to exercise curiosity, critical thinking, and creativity in an active, collaborative learning environment. Our teachers demonstrate expertise in their fields and a passion for lifelong learning." Ms. Johnson has been given her own classroom and wants to ensure that the classroom setup promotes active, collaborative learning, as described in the school mission statement. Which of the following classroom setups would be most appropriate?

tables that form small groups

Mrs. Leon finds a cell phone that she believes belongs to one of her students, Jeremy. Just last week, Jeremy's phone was taken away in class after he had texted a classmate and disrupted the class. Mrs. Leon suspects that Jeremy's message might have been an inappropriate comment about her. What should Mrs. Leon do?

take the cell phone to the administrator's office to be picked up by Jeremy

Ms. Drucker is a very creative teacher. She is always coming up with ideas for new ways to teach concepts and new activities to try with her English Language Arts students. She has a new activity planned for students almost every day. A potential problem with this approach is that introducing new activities:

takes more class time than doing things in a familiar way.

All of the following are ways that teachers can create a climate of high expectations except:

taking a directive role in planning activities, providing little opportunity for student choice.

During a lesson on China, a student remarks that they saw something on the news about technology developments in China. The teacher briefly pauses the lesson and the class discusses the news story and the impact it has on the students. What is this teaching strategy called?

teachable moment

A preschooler has two breakable graham crackers. The child states that when he breaks the two into four separate but equal crackers, he now has more graham crackers. The best assessment of this child's thinking is:

that he has not grasped the concept of a whole and its parts.

A teacher is developing an assessment to determine what students have learned during the semester. Which of the following is the most important for the teacher to consider as she develops this test?

that the test is carefully aligned to the material taught throughout the semester

Which of the following is the most appropriate professional development activity for teachers?

university classes or professional workshops relating to the field they teach

An elective teacher has concerns about a student and asks the school counselor to see the child's file. The school counselor checks that this student is in the teacher's class and asks the teacher to sign an access log. Which of the following federal mandates protects the child's file?

the Family Educational Rights and Privacy Act (FERPA) of 1974

Which of the following should not be considered when evaluating an internet source for credibility on a topic?

the amount of relevant information provided by the source

On the first day of class, Mrs. Patterson wants to ensure she creates the best classroom environment for her third-grade students. One of the first things she should convey to the students is:

the classroom rules and procedures, letting them know she expects each student to follow the rules.

When asking a student a question, which of the following should be the primary consideration of the teacher for response time by a student?

the difficulty level of the question

Mr. Shaf is known as a great technical building teacher at Pawnee High School. He has decades of experience and his knowledge surpases any other instructors in the district. He is know as a difficult teacher and students claim that he does not like for students to ask questions. A new assistant principal (AP) reviews one of his classes and states that he needs to improve his listening skills with students and should evaluate the way he responds to questions. This critique most likely means:

the new AP is trying to help Mr. Shaf improve the climate of the classroom to allow students to feel emotionally safe enough to ask questions and improve their access to learning.

A high school biology teacher, Mrs. Adams, wants to work with the school groundskeeper to coordinate a project for her students that promotes environmental awareness. In working with the groundskeeper, it is most important for Mrs. Adams to be able to identify:

the objectives and goals of the project and how the groundskeeper can help achieve these goals.

Mrs. Love is a seventh-grade mathematics teacher. Her students continue to ask how the material on determining slope will apply to their lives outside of school. Of the following, what would be the most effective resource to teach about the practical uses of mathematics outside the classroom?

the stock market page from the local newspaper

In this modern day of nontraditional households, the IEP committee must fully understand and interpret the meaning of the word "parent." A parent can be considered any of the following except:

the student's long-term babysitter or nanny

Mrs. Baldor promotes learning when introducing her eighth-grade English class to a short story about fear by asking her students to brainstorm their thoughts on the word "fear." She then lists all their responses on the board. Her reasoning for using this open-ended strategy can best be explained by:

the use of a motivational technique promoting engagement due to common knowledge and experiences, plus the acceptance of all responses.

Use the conversation between Daniel, a high school English-language learner (ELL), and his teacher to answer the questions that follow. Mr. Jedson: What are some ways we can stay healthy? Daniel: We can to exercise and eat healthy foods. Mr. Jedson: Yes, and what else? Daniel: Well, we shouldn't smoking or drinking too much alcohol. Based on the grammatical errors that Daniel makes in the conversation, which of the following topics would he most benefit from a lesson on?

the use of modal verbs

After learning about the importance of providing young children with a broad understanding of a new concept, a second-grade teacher is evaluating how to incorporate interdisciplinary learning into her classroom. Which of the following will best support this teacher's instructional goal?

thematic units

A kindergarten teacher notices that every day during free play time, many of the children choose to play with the dump trucks and diggers in the sandbox. With this knowledge, she decides to devote a week to construction equipment. During this week she reads the children stories about the different types of construction equipment (dump truck, backhoe, etc.), practices basic addition and subtraction with dump trucks and blocks, and has children experiment with mixing sand, rocks, and water. In doing this, the teacher is demonstrating her understanding that:

thematic units are an effective way to integrate different subjects.

In order to help students integrate information and make it easily available for recall, teachers should plan:

thematic units that include all core subjects.

Thoughtfully prepared lesson plans should include meaningful and authentic L2 activities that integrate concepts (letter writing, models, simulations) with opportunities to practice language (reading, writing, listening, and speaking). These are essential components of an ELL instructional plan because:

they support students' content mastery and L2 proficiency.

Sayville High School offers a science camp every summer to students aged ten and older. When designing the activities to complete during the camp, the instructors must keep in mind that the younger students might not have the cognitive ability to do which of the following?

think abstractly about a concept

Use the information below to answer the questions that follow. The information presents a hypothetical classroom scenario, and the questions require that you make the best decision, based on the presented information. Ms. Wright is a 6th-grade math teacher who likes to design her own task-based lessons to go along with the school curriculum. For a unit on statistics, she designs a project in which students create statistical questions, survey others, display and analyze the results and report their interpretations and conclusions in a presentation to the class. After each step, Ms. Wright has the students turn in their work so that she can identify any errors and provide constructive feedback. When the students present their conclusions to the class, Ms. Wright is impressed by her students' presentations and findings but worried because the task has taken two days longer than it should have. Now, she feels pressure to cut down the content in the next unit to get back on schedule. Ms. Wright tells her mentor teacher about her concern, and together they list the specific problems that were causing her delay: 1) Too much time was spent collecting student work and returning it to students. 2) A lot of time was spent commenting on student work and providing feedback. 3) Students often misplaced parts of their assignment, as it was constantly being transferred back and forth from student to teacher. In order to avoid these issues in the future, Ms. Wright's mentor is likely to suggest some tips related to:

time and material management.

Before reading a text on the food web in a 5th-grade science class, Ms. Steiger provides students with some general statements to elicit a response. She asks students to either agree or disagree with the statement and if possible, explain their reasoning. After reading the text, students return to the statements and indicate if their opinions have changed or were strengthened. The purpose of this activity is:

to activate students' prior knowledge.

After attending a classroom management workshop, a teacher realizes that her current strategies are not effective. Her most appropriate next step is:

to ask an administrator for help in reforming her classroom strategies.

At the beginning of the year, Ms. Wright sends home a letter and a soft-copy via email to parents, detailing the classroom expectations, academic overview, and the best means of communicating with her through the year. What is the purpose of this document?

to begin the year with positive and clear communication and to begin building a relationship with parents

Formative assessments provide information that can be used for which of the following?

to change and improve instruction

At the start of every school year, Mr. Brown writes his students a letter telling them about himself, and he asks them to write him back as their first assignment. The main purpose of this activity is:

to develop a trusting relationship with students.

Read the information below and complete the questions that follow. Every week Mr. Brown devotes an entire class period to small group activities. He separates the students into heterogeneous groups, which change each week so that students have the opportunity to work with every classmate at least once. At the end of the month, he reflects on the positive and negatives of group work and creates the following list: Positives Negatives Exposure to different perspectives Improved ability to cooperate with others Enhanced social skills and ability to interact with others More student-centered approach Unequal participation - Some students are passive participants in the group. Can be time-consuming Occasional conflict among group members Mr. Brown always separates the students into heterogeneous groups. For which of the following reasons would it be best to group the students homogeneously instead?

to differentiate and focus on specific skills

Which of the following best describes the purpose of a scope and sequence?

to ensure that the teaching of instructional concepts and goals progress logically

Use the information below to answer the questions that follow. The information presents a hypothetical classroom scenario, and the questions require that you make the best decision, based on the presented information. Ms. Wright is a 6th-grade math teacher who likes to design her own task-based lessons to go along with the school curriculum. For a unit on statistics, she designs a project in which students create statistical questions, survey others, display and analyze the results and report their interpretations and conclusions in a presentation to the class. After each step, Ms. Wright has the students turn in their work so that she can identify any errors and provide constructive feedback. When the students present their conclusions to the class, Ms. Wright is impressed by her students' presentations and findings but worried because the task has taken two days longer than it should have. Now, she feels pressure to cut down the content in the next unit to get back on schedule. Ms. Wright tells her mentor teacher about her concern, and together they list the specific problems that were causing her delay: 1) Too much time was spent collecting student work and returning it to students. 2) A lot of time was spent commenting on student work and providing feedback. 3) Students often misplaced parts of their assignment, as it was constantly being transferred back and forth from student to teacher. What is the primary reason that Ms. Wright plans to meet with a mentor teacher?

to improve her instructional approach in the future

The purpose of handing out a course syllabus at the beginning of the semester is:

to inform all students of the course curriculum, expectations, and assignment due dates.

What is the primary reason for administering an achievement test?

to measure skills gained over a period of time

Ms. Johnson has implemented small, homogeneous, intervention groups in her class to provide instruction to students who have similar needs. This allows Ms. Johnson to invite individual students to join the group to work on a specific set of skills. Which of the following would be the best use of these intervention groups by Ms. Johnson?

to provide instruction to a minority of students who have not demonstrated mastery of a specific content objective

Mrs. Vazquez is a seventh-grade history teacher with students from a wide geographic area. For a class project, she has her students write their state representatives asking them to come visit and speak at their school. Out of the three representatives contacted, two wrote a letter thanking the students for their time but declined the request, and one representative made a visit to the campus. The most likely benefit for the students in writing the letters is:

to show the students that they were taken seriously by elected officials, which could encourage them to engage in political activity in the future.

Students who are taught to tie their own shoes can then tie any pair of shoes without being retaught. This cognitive ability is known as:

transfer.

A prekindergarten teacher has several songs she sings throughout the school day with her students. They are called, "Circle Time," "Clean Up," and "Goodbye, Friends." Including these songs in the daily class schedule helps the students:

understand behavioral expectations and transition between activities smoothly.

Mrs. Crow provides her students a self-assessment rubric to complete after a semester-long group research project. Which of the following does a self-assessment rubric best promote among students?

understanding and taking responsibility for their learning

Which of the following is the explanation of learning theory?

understanding how people learn and incorporate new information into their schema

During lunch, there are four teachers in the lounge eating. One teacher begins discussing a student's academic issues, which she believes are affected by the student's home life. This discussion is considered:

unethical because it breaches privacy laws.

Ms. Johnson, a sixth-grade social studies teacher, decides to include more group work in her class assignments. She places students in groups of three for their "Cultures of Europe" projects. The student groups need to research a topic, write a paper, and make a presentation to the class. The presentation must engage the participation of the whole class, include a summary handout, and utilize technology. The groups must also document every member's participation in all parts of the project because each group will receive a single grade. The purpose of this assignment is to:

use student-centered cooperative learning activities to increase student learning and positive social interactions with peers.

As Mrs. Matthews, an ELAR teacher, grades a required composition, she writes a note concerning each of her students' work. According to research on instructional feedback, which elements would be the best method for providing feedback to students?

using an established rubric with further notes of the specific positives and informing the student about the specific errors that need to be addressed

Which of these is a verbal communication strategy being used by a teacher?

using different voices to convey characters in a story

Which of the following is the most effective way for a teacher to use technology to increase their productivity?

using real-time collaboration and authoring tools to share documents with students and edit student writing assignments

Mr. Prince teaches fourth-grade math. He records his lessons and posts them online for students to watch when they need to review. He explains verbally and writes down instruction. He also offers manipulatives for the class to use when needed. Mr. Prince's way of teaching is a great example of:

using varied strategies and adjusting communication to reach all students.

Which of the following best describes the degree to which an assessment accurately measures what it intends to measure?

validity

Which of the following terms means that a test measures what it is supposed to measure?

validity

Which of these choices is NOT an important consideration in increasing the effectiveness of feedback?

verbal vs. non-verbal

A new teacher is planning a unit of study. Which of the following should the teacher first consider when planning for instruction?

what will be the main concepts or skills addressed during the unit

All of the following are examples of effective teacher modeling except:

writing a description of the task on the board


Kaugnay na mga set ng pag-aaral

Environmental Studies (Chapter 2)

View Set

Modalidades de la conciencia De acuerdo a la ley moral Conciencia:

View Set

ap world history unit 1-4 questions

View Set

Abeka 11th grade Grammar V Quiz 17

View Set

s66 quizzes: federal securites acts

View Set

Insurance Regulations - License, State Regulations

View Set